You are on page 1of 111

adsfadsfasdfadsfrecepters, -2 q's 36. endocarditis-viral, bacterial 37.

India ink
-crypto 38. ida, anemia of chronic dz 39. vwb dz- whats asdfasdfadsfasdfrnit wel
l ,2q's 69. myasthenia gravis- 2 q's 70. hashimotos= 2 q's 71. dermatomes 72. 68-10, 8-10-12, lung -pleura relationship 73. rt. Atrial hypertrophy on cxr 74. p
aradoxical emboli 75. brca met. via axillary nodes 76. soft tissue sarcoma 77. h
emoglobin c 78. hla b27- reiters syn 79. proteus- uti stones 80. pathogenesis of
gall stone-supersat. Of cholesterol 81. acute panreatitis vignette-inc. amylase
82. nodular glomerulosclerosis in D.M. 83. know your vaccines, and active and p
assive immunizations 84. cohort study, case series 85. I had 5 q's on epidem. (p
pv, spec, sens) 86. p=id 87. endometriosis 88. 2 bitemp. Hemianopsia's 89. know
your goiters- toxic, hot, cold, non toxic, multitoxic 90. brached hyphi- dermato
phyte 91. hyper -PTHism 92. cushings syn 93. addisons dz 94. duchenne's musc. Dy
strophy- pedigree 95. osteo sarcoma 96. comparing osteomyleitis and osteo chondr
itis- weird q 97. pseudo gout 98. neuro syph- arguile robinson pupil 99. vitamin
s- b12, pyridoxine, and vit c 100. desmopressin in diabetes insipidus 101. know
your murmers. How thy look normally and on a wiggers 102. I had no ekg-s 103. sa
lmonella 104. mhc 1- tap ptotein 105. cavernous hemangioma, is it bad? 106. lot
of angio tension -renin stuff- 4 q's 107. ipratropium in copd 108. colchicimein
acute gout 109. filtration fraction up and down..why? 110. know your acid-base s
tuff-adsfadsfadsfasdflide of mammilary bodies (pertaining to chronic alcoholic)
extraocular muscles and innervations many, many questions on blood supply as the
y pertain to stroke presentations (including several brain arteriograms) many, m
any vision questions (know it cold) several B12 neuropathy questions several bra
chial plexus questions muscle spindle control Behavadsfadsfadsfdiuretic use hydr
ochorothiazide causes hyperglycemia K+ sparing diuretics--spirinolactone can cau
se gynecomastia quinidine several questions on lipid-lowering agents anti-cancer
drugs were big too: 5 fluorouracil, doxorubicin, prednisone, raloxifene (make s
ure you know all their side effects) Magnesium antacid abuse heparin/warfarin ma
ny NSAID questions glucocorticoids propylthiouracil Physiology many heart questi
ons with graphs preload vs afterload resistance, pressure and flow capillary flu
id exchange surfactant function many Hb dissociation curve questions somatostati
n vagotomy effect on digestion

bilirubin, bile glucose clearance in kidney several PTH/calcitonin/Vitamin D que


stions ANP many renin-angiotensin questions know hypothalamic/pituitary regulati
on well many estrogen/progesterone/menstruation/pregnancy questions spermatogene
sis

Step 1 Questions I remember: 1. MOA of rifampin (darn question showed up 2 times


) 2. MOA of erythromycin 3. MOA of cephlosporins 4. What do u give first to unco
ncious alcholic who comes in to ER? IV thiame first 5. You have bad news on pati
ents condition and have to tell her, she see statement on your face and says son
makes all the decisions, what do u do? Tell her, Tell her and son, Tell son, Te
ll nurse to tell son, ect... 6. 16 yo girl comes in and has bf and they have pro
tected and unprotected sex, what do u tell her first in regard to contraception?
7. Cystic Fibrosis kid, what chromosome is the defect on? 8. Philadelphia chrom
osome found in what condition? 9. What do u see on EM with a kid with minimal ch
ange disease? 10. Athelete suddenly collapses while playing baseball, what kind
of cardiomyopathy? 11. Rate limiting step in glycolysis? 12. Weber - Rinne test
of person with cerumen in his right ear what happens? 13. Weber - Rinne test wit
h person who is deaf in R ear what happens? 14. Corneal reflex question with R e
ye have consenual reflex only and when with L eye, direct only, where is the pro
blem? 15. Cross section of spinal cord asking location that is affected with los
s of pain and temp. 16. Cross section of spinal cord asking location that is aff
ected with UMN lesion. 17. CT of brain of person involved in car accident, what
kind of hematoma? Subdural, epidural or subrachnoid? 18. Enlargement of lateral
ventricles with blood in them, where is the problem? 19. Brachial plexus drawn a
nd sking for where lesion is with Erbs? 20. X-Ray of femur asking location where
avasular necrosis occurs? 21. Drawing of chest asking where u hear the mirtal v
alve sound the best. 22. Pressure volume loop curves asking where mitral valve o
pens? 23. Diagram with normal heart sound and abnormal one asking what kind of m
urmur is seen on the abnormal? It was a late diastolic murmur. 24. Side effects
of combo drugs of person being treated with Hodgkins? 25. If person has megalobl
astic anemia with folate and neuro problems and after Rx u only have neuro probl
ems what did u give patient?

26. Question on Fe deciciency anemia, what values u see with TIBC, seurm Ferriti
n, etc. 26. Person with diabetes insipidus what lab values do u see? 27. Person
with SIADH what lab values do u see? 28. How do u distinguish between an insulim
ona and person who is taking insulin for diabetes? 29. What cardiovascular probl
em is seen in a patient with Marfan's syndrome? 30. What cardiovasular problem i
s seen in patient with teritiary syphilis? 31. What problems with eyes is seen i
n patient with teritiary syphilis? 32. MC organism seen in hikers who drink wate
r from mountain stream? 33. Know the steps in Gram staining and for acid fast st
aining. 34. What is vaccine for H. Influ b made from? 35. How do u calculate sen
sitivity? 36. How do u calculate positive predictive value? 37. How do u calcula
te attributable risk? 38. What is a cohort study? 39. What is a double blind stu
dy? 40. How do u calculate prevalance? 41. In what situation is prevalance great
er than incidence? Chronic diseases 42. What happens to GFR and RBF if there is
constriction of efferent arterioles? 43. Lineweaver Burk plot asking what happen
s with competitive inhibitor? 44. Michaelis Menton curve asking what happens wit
h a competitive inbibitor? 45. What kind of mutation happens when there is chang
e from long protien being made to short one? 46. Know about Wobble hyopthesis 47
. Where is mRNA made in the cell? 48. MOA of erythromycin? 49. Family members al
l vomit 5 hours after eating BOILED rice that sat overnight with NO refrigration
? B Cerus or ETEC 50. SE of person taking aminoglycosides? 51. How old is child
when they don't have stranger anxiety? 52. Characteristics of person suffering f
rom bipolar disorder? 53. Characteristics of person suffering form schizophrenia
? 54. Parent bring kid in who was unconcious, he has nystagmus and has been with
drawn from parents, what drug has he been abusing? 55. Person who is taking hero
in can have what kind of infection? 56. Organism causing diarrhea in person with
AIDS? 57. Organism causing diarrhea in infants who go to daycare?

58. What kind of pattern seen on EM of patient with Goodpasteurs? 59. Picture of
vegetations on aortic valves, what kind of problem will they cause? 60. Know di
fference between R sided heart failure and L sided heart failure 61. Person is b
ulemic, what is PCO2, HCO3 and pH? 62. If go to high altitude what is compensato
ry mechanism of respiratiory alkalosis and what are they values u would see PCo2
, HCO3, pH. 63. Person with H.Pylori given medication that covers ulcer, what is
MOA of that drug? 64. SE of person taking H2 blockers? 65. Best treatment of pe
rson who is Black suffering from CHF? 66. Give heparin to person but no change i
n PT or PTT what is deficient? 67. What changes in persons PT, PTT and Bleeding
time with von Willebrands disease? 68. Same as above but now person with hemophi
lia A? 69. Know steps involved in doing a PCR. 70. Person comes in with a magazi
ne ad or a new drug which you as a doctor know won't help them in their conditio
n and patient asks if you can give them the drug, what is first thing you tell t
he patient? 71. 13 yr old kid comes in who admits to being sexually abused by st
epfather, what is the first step you take? Notify child services or call police
or talk to mom or talk to step father

Cases/questions: Female patient is on a whole bunch of drugs including Warfarin,


diuretics, antihyperlipedimics, etc. She has recently become pregnant and has c
onsulted the doctor, which of the drugs should be discontinued. Answer: Warfarin
(teratogenic for fetus). Which of the abdominal aorta branches supply the foreg
ut Answer: Celiac trunk Patient presents with inflammation and necrosis of small
intestine, ascending colon and proximal 2/3 of transverse colon, which blood ve
ssel is involved: Answer: superior mesenteric A. Which branchial arch makes up M
andible and Mylohyoid, etc. Answer: first arch Picture of endochondral bone form
ation (similar to picture in first aid) and asks which of the following bones ar
e formed by the shown process. Choices were clavicle, skull, femor, etc. Answer:
Femur (long bones) Picture of a child being able to bring her shoulders togethe
r, which bone is she missing: Answer: clavicle Which cells in testes provide the
Blood Testes barrier: Answer: Sertoli cells Knee injury and tear in MCL and ACL
Which organ in the body releases most ANG II into the body? Answer: lungs (has
ACE) EM of muscle asking to point to Z line Scraping the stratum corneum from th
e skin, what is the most important complication? Answer: loss of fluid from body
and dehydration (since stratum corneum contains Keratin and protects against bl
ood loss) Gap junctions are able to transmit Electrical current between cells

Patient presents with fever and shivering. What is the purpose of shivering? Ans
wer: thermoregulation (it generates heat). Sudden wild flailing of one arm, wher
e is the lesion of the drawing: Answer: subthalamus Patient presents with deviat
ion of uvula and shoulder droop, where is the lesion? Answer: jugular foramen Po
int to the muscle that depresses the eye from the adducted position Answer: Supe
rior Oblique muscle (same diagram as in page 108 first aid, 2001 edition). Marcu
s Gunn Pupil Lesion in which blood vessel causes left homonymous hemianopsia (an
giography similar to circle of willis pictures in High Yield Neuro) Answer: righ
t post. Cerebral Brown Sequard lesion Dorsal Column pathway and lesions of Graci
lis fibers and Cuneatus fibers and how they affect sensation on upper and lower
limbs. Radial nerve injury Cervical rib and what it affects Answer: inf. Trunk o
f brachial plexus Cohort study Heroin addiction and long term maintenance Answer
: methadone Delirium (patient not oriented as compared to dementia in which pati
ent is oriented) Conversion disorder Post traumatic stress disorder Personality
disorders: Avoidant and Schizotypal

Narcolepsy Borderline patient using splitting as a defence mechanism (either eve


rything is good or everything is bad) Lots of questions on shizophrenia (at leas
t 4) Thought disorder, Loose associations, etc. Hardy Weinberg population geneti
cs calculations Glucose-6-phosphate dehydrogenase deficiency and reaction to sul
fa drugs Which a.a. become essential in a patient with PKU with phenylalanine re
stricted diet: Answer: tyrosine Picture of patient with blue sclera Answer: abno
rmal collogen synthesis Patient presents with muscle weakness and hypertrophy of
calf muscles, which chromosome is defective? Answer: X chromosome (DMD) Uncoupl
ing agents and on which part of mitochondria they work? Anwer: dissipate proton
gradient across the inner mitochondrial membrane Urea cycle Arachidonic Acid pro
ducts (very heavily tested, at least 5 questions) LT B4 (neutrophil chemotactic
agent) LT C4, D4, E4 (slow reacting substances of anaphylaxis) TX A2 (vasoconstr
ictor and stimulates platelet aggregation) PG I2 (inhibits platelet aggregation
and vasodilation) Collogen Synthesis and structure Vitamins (heavily tested): B1
(alcoholics)

B2 (cofactor in oxidation and reduction-FADH2) B6 (given along with isonizid to


prevent peripheral neuropathy) Vitamin C (hydroxylation of proline and lysine in
collogen) Vitamin D (which organ hydroxylates 25 hydroxy D3 at alpha1 position:
kidney) Vitamin K (coagulation factors) Which test diffentiates staph aureus fr
om other staph (believe it or not!) Answer: Coagulase test Alpha Hemolytic bacte
ria H. pylori infection (at least 2 questions) BMT regimen Which of the pairs re
sult in long term neurological complications? Answer: Borrelia and Treponema Muc
or infection in diabetics (same picture shown as on page 191 of first aid, 2001
edition) After an earth quake in california, epidemic of lower respiratory infec
tion which lasts for 2-3 weeks, which of the bugs responsible for it? Answer: Co
ccidiodomycosis Candida vaginalis in diabetics Toxoplasma and brain abcess in HI
V Giardia and foul smelling stools Most important sequale in infectious Mono: An
wer: splenic rupture (I think). Patient has C3 deficieny, which disease is he mo
st likely to be susceptible to Answer: only capsular organism listed was pnomoco
ccus 2 questions on Huntington Disease:

trinuclear repeat pattern point to Caudate Nuclei on MRI alpha fetoprotein as a


marker for malignancy 11,22 translocation in Ewing sarcoma of the bone
Hereditory Spherocytosis and treatment Answer: splenectomy IgG spike on plasma p
roteins (shown in a diagram) and asked how the patient presented Answer: bone pa
in Hirschprungs disease and what causes it Patient presents with cirrhosis after
being diagnosed of liver disease many years ago. When he presented at first ALT
was elevated, what is the most likely cause of cirrhosis? Answer: Hep C (if AST
was higher, it would have been alcoholic cirrhosis). Chronic pancreatitis and it
s association with alcoholism Pancreatic adenocarcinoma and signs associated wit
h it (eg. Weight loss, obstructive jaundice and palpable gallbladder Diagnosis o
f emphysema in a non-smoker 25 year old with dyspnea, hyperexpanded lung fields
and lowered breath sounds and its association with a1antitrypsin deficiency (pana
cinar emphysema) Restrictive lung disease and FEV/FVC ratio (80%) Paraneoplastic
syndromes associated with small cell carcinoma of the lung and the signs associ
ated with it (eg. Hyponatremia associated with SIADH in small cell carcinoma) Pa
tient presents with tremor and rigidity, upon taking history you find out that p
atient has been experiencing with a designer drug. What is the most likely regio
n of the CNS thats involved Answer: Dopaminergic neurons of substantia nigra Trea
tment of Parkinsons Patient presenting with upper and lower motor neuron problems

Answer: ALS Epidural and Subdural hematomas and their diagnosis based on history
. Patient presenting with signs of Brocas Aphasia, point to region of the brain t
hat is most likely involved. Diagnosis of Horners Syndrome Patient presents with
signs and symptoms of arthritis, morning stiffness and Dup. Deformity of the han
ds and fingers, what abnormal test result is expected. Answer: Anti-IgG antibodi
es in the serum (diagnostic of RA) At least 3 questions on gout and drugs used f
or it (eg. Patient presents with joint pain after chemotherapy for Hodgkins lymph
oma, etc.) SLE and kidneys wireloop lesions, False positive on syphilis test (VRD
L) and drugs which induce SLE and diagnostic features Patient presents with Sacr
oiliac joint inflammation, which HLA tests are likely to be positive Answer: cho
ose whichever one had HLA B27 Hyperaldosteronism and its diagnosis based on renin
, Na and K values in plasma Distinguishing between primary and secondary adrenal
insufficiency Pheochromocytoma and drugs used for it (esp. nonselective irrever
sible alpha blocker) All the features associated with MEN I and MEN II Hypho and
Hyperthyroidism and diagnosis based on symptoms and signs and plasma values of
T3, TSH for each Diabetes types I and II (at least 5 questions)

Ketoacidosis associated with type I Symptoms associated with diabetic coma Renal
complications (arteriosclerosis and glomerular lesions on light microscope Lab
values associated with diabetic ketoacidosis
Carcinoid tumor diagnosis based on symptoms and what abnormal test is positive i
n urine Answer High HIAA in urine

Osteoprosis and its association with corticosteroids BPH and drugs used for it Hy
adid mole and increased HCG associated with it early in pregnancy Endometrial hy
perplasia and incrased risk of endometrial carcinoma with abnormal uterine bleed
ing An obese female presents with high LH and hirsutism. Dx? Answer: Polycystic
Ovarian Syndrome Cardiomyopathies esp. Hypertrophic cardiomyopathy: hypertrophy
involves the interventricular septum and 50% is caused by mutation in B globin g
ene of heavy chain (familial). Usually seen in athletes. Heart murmurs and their
diagnosis based on features of the murmur DVT and pulmonary embolism. Bone marr
ow embolism seen as a complication of bone fractures. May lead to sudden death.
Wegeners granulomatosis and its Dx based on symptoms (C-ANCA was not given). WG in
volves the upper airways and youll see granulomas (this is the distinguishing fea
ture from Goodpastures syndrome Minimal change disease effacement of BM Acid Base
physiology Pharmacodynamics graphs distinguishing Agonist with competitive anta
gonist and irreversible antagonist Mechanism of Tetracycline Nephrotoxicity asso
ciated with Aminoglycosides and also ototoxicity. Isoniazid and B6 which can pre
vent neurotoxicity HIV triple therapy (two nucleotide reverse transcriptase inhi
bitors with a protease inhibitor. Mechanism of Dantrolene Isoprotrenol effect an
d reflex tachycardia associated with it Mechanisms of action of sympathetic rece
ptors, Dopamine receptors (eg. cAMP, IP3, etc.)

Antipsychotics and Tardive dyskinesia Lithium and its effect on thyroid (may caus
e hypothyroidism) Sumatriptan and its mechanism of action ACE inhibitors and its e
ffect on blood pressure and its mechanism of action Cardiac glycosides and its eff
ect on intracellular and extracellular Na and K Busulfan associated with pulmona
ry fibrosis Cisplastin associated with Nephrotoxicity Lead poisoning and its feat
ures (eg. lead lines) and treatment H2 Blockers and their mode of action and the
ir effects on gastrin and stomach acid Arachidonic acid products and their effec
ts on vascular tone and platelet action Leuprolide and its indications (also mech
anism of action). Pacemaker action potential and its differences with ventricular
action potential (eg. upstroke is opening of Ca channels rather than Na channel
s). Calculation of cardiac output with Fick principle Cardiac cycle graphs Mecha
nism of smooth muscle contraction and its differences with skeletal muscle (eg.
involvement of calmodulin) Capillary fluid exchange and calculations involving h
ydrostastic and oncotic pressures, etc. Glucose clearance and at plasma glucose
of 200 glucosuria begins PTH and Vit D effects on plasma Ca and phosphate. Stero
id hormone mechanisms (binds to receptor located in nucleus or in cytoplasm) Dis
tinguishing 11B hydroxylase and 21B hydroxylase Prolactin regulation and Dopamin
e Oxygen dissociation curve and which factors effect it (eg. pH) Secretin and st
imulation of release of antacid

Bile and how its recycled (absorption via ileum and how it causes malabsorption o
f fat in patients with ileal resection). Here are the q s I recall: 1. know your
mediators (IL-2, IL-456, IL 10, TNf,TGF, IFN) 2. know cd 16 and 56 are killer c
ells 3. where both clotting pathways meet 4. acute MI what the gross and histo l
ook like at all stages, 5. brachial plexus- I had ulnar nerve b/c interossei wer
en t working 6. know 2nd messengers-oxtyocin, alpha 1 7. herpes pneumonitis, wit
h giant cells 8. splenic infarct, from where? 9. cardiac tamponade 10. go over j
amie s sheets- 10 icm type q s 11. know the arachadonic acid pathway and enzymes
and inhibitors 12. t(9;22) cml 13. lat rectus on mri 14. wilsons- they showed t
he kleiser fleicher ring 15. cystic hygroma- turners syn 16. mitochondrial inher
itance -2 q s 17. know everything about digoxin 4 q s 18. pku 19. b-cell def- no
germinal centers in LN 20. know the cyto p450 inducers and inhibitors - 6 q s 2
1. scleroderma- ab against? 22. I had no sle 23. bph and prostate cancer- drugrs
, recepters, 3q s 24. ebv in Aids cancer- lymphoma 25. know your endocrine physi
o- hormones up and down 26. know the resp. curves in first aid, where would fibr
osis be? 27. know histo of the hepatocyte- fxn of organelleI had mitochiondria 2
8. know histo of kidney- pct, dct, loh- where glucose is reabsordeb, where vit d
is activated 29. restrictive cardiomyopathy 30. p. carinii -know what it looks
like 31. marasmus 32. carcinogens in bladder cancer 33. schistosomes in bladder
cancer

34. ASCVD-2 q s 35. prinzmetal angina- drugs, recepters, -2 q s 36. endocarditis


-viral, bacterial 37. India ink-crypto 38. ida, anemia of chronic dz 39. vwb dzwhats up? 40. henoch-scholheim purpura on the leg-vasculitis 41. pathogenesis o
f TB 42. didn t have a lot of behavioral but what I did have was alzheimers, and
a few "what do u tell the pt" q s 43. hand foot mouth dz- coxsackie 44. cherry
red macula and European-hexoaminidase a2 45. gauchers 46. tuberous sclerosis- 2
q s (when did we learn this)- know it is assoc. w/ embryonal rhabdomyosarcoma an
d calcifications in brain 47. cancer met. to colon 48. crohns dz 49. hirshprungs
dz- failure of neural crest cells 50. know your gi physio- 5 q s 51. moa of ome
prazole 52. moa of vancomycin 53. moa of neostigmine 54. causes of malig. Hypert
hermia 55. pathogenesis of spider angioma 56. malig. Htn- pic 57. min. change dz
- vignette was classic 58. pid-chlamydia 59. carnitine def-s/s 60. transformatio
n- dnase 61. fracture to fibula- loss of deep peronial nerveloss of dorsiflexion
2q s 62. damage to base of penis-blodd flow is lost in which part of urethra (we
ird q) 63. hypertinic bladder-whereis lesion? 64. where does seminoma drain? 65.
know your fractures- what muscles pull them where- 3 q s 66. fracture to lt. 911 ribs- spleen 67. cavernous sinus thrombosis- fxn s lost 68. av nicking , bank
ing- learnit well ,2q s 69. myasthenia gravis- 2 q s 70. hashimotos= 2 q s 71. d
ermatomes

72. 6-8-10, 8-10-12, lung -pleura relationship 73. rt. Atrial hypertrophy on cxr
74. paradoxical emboli 75. brca met. via axillary nodes 76. soft tissue sarcoma
77. hemoglobin c 78. hla b27- reiters syn 79. proteus- uti stones 80. pathogene
sis of gall stone-supersat. Of cholesterol 81. acute panreatitis vignette-inc. a
mylase 82. nodular glomerulosclerosis in D.M. 83. know your vaccines, and active
and passive immunizations 84. cohort study, case series 85. I had 5 q s on epid
em. (ppv, spec, sens) 86. p=id 87. endometriosis 88. 2 bitemp. Hemianopsia s 89.
know your goiters- toxic, hot, cold, non toxic, multitoxic 90. brached hyphi- d
ermatophyte 91. hyper -PTHism 92. cushings syn 93. addisons dz 94. duchenne s mu
sc. Dystrophy- pedigree 95. osteo sarcoma 96. comparing osteomyleitis and osteo
chondritisweird q 97. pseudo gout 98. neuro syph- arguile robinson pupil 99. vit
amins- b12, pyridoxine, and vit c 100. desmopressin in diabetes insipidus 101. k
now your murmers. How thy look normally and on a wiggers 102. I had no ekg-s 103
. salmonella 104. mhc 1- tap ptotein 105. cavernous hemangioma, is it bad? 106.
lot of angio tension -renin stuff- 4 q s 107. ipratropium in copd 108. colchicim
ein acute gout 109. filtration fraction up and down..why? 110. know your acid-ba
se stuff- weird q s with combined.alcoholic that vomits and had diarrhea (what t
he !@#$#$?) know the values 111. know your lesions in the CNS- whats ipsi whats

contra 112. little biochem on the exam-know the rls s 1. I had two questions r/t
Baye s Theorum (Genetics) 2. Remember how I said ANS &CNS Pharmacology are Key!
! 3. Gliaden and wheat causing Celiac Sprue 4. They showed a picture of a Penis
with a lot of Painful nodules. They did not have H. ducreyi, T. pallidum, Herpes
, as the choices!!! By the way, this was the first question I saw after I got ba
ck from Lunch. 5. Conditions favoring Metabolic Acidosis/Alkalosis 6. Slide of H
airy Cell Leukemia. 7. I had five Questions on Lyme Disease (Go Watsons!) A word
of Caution, Lyme Disease does happen in Oregon. 8. What Causes Trendelenberg Si
gn and Waddling Gait 9. Know all about Heparin and Warfarin 10. I had a question
on Croup and Slapped Cheek Disease (Fifth Disease) 11. Lots of Vitamin Question
s, so know the roles of Vitamin A,C,K,B12,B6, Thiamine. 12. For the behavioral q
uestions, they would ask you what the next step is. Pick the least invasive and
the answer that gives the most degree of freedom. What I mean is that if the com
petent patient refuses to comply with the doctor, that is his right. 13. For tho
se that have the plagarized MSA Notes (Red ICM Book), it would be a good idea to
go over them before the exam. I had a few respiratory questions that could not
have been answered had I not red the chart in the book. 14. Role of Gap Junction
s 15. Nephritic and Nephrotic Syndromes 16. I only had one SLE Question 17. McAr
dles Disease, Tay Sachs, Gaucher s Disease were questions. ANATOMY -mesonephric
ducts=male spermatochord -inguinal fascia and saddle injury (kids starts to pee,
where will urine go inside body cavity, NOT down thigh) -diaphragmatic hernia GI blood supply (celiac trunk=forgeut) -ureters and obstructions -macula adheren
s -ciliar dynein (Kartagener s syn) -picture of lesion in Broca s area -spianl c
ord lesions -braichial plexus (of course, mine was axillary n.) BEHAVIORAL -lots
of "how would you respond as the physician" questions, WITH lengthy clinical de
scriptions. read the actual question first, it will allow you to skip a lot of t
he data they

give -graph asking how you would maximize sensitivity -odds ratio -types of stud
ies (cohort, case-control) -speech development (what the hell?) -developmental m
ilestones -narcolepsy -sleep apena (central vs. obstructive) X2 BIOCHEM -DNA mut
ations -where in cell introns are removed -tRNA attachemnt to ribosomes -modes o
f inheritence (I actually had one ask me the chance of a MAN passing an Xlinked
recessive gene to a son, read CAREFULLY) -urea cycle -carnitine -glucokinase vs.
hexokinase kinetics -enzyme kinetics -collagen structure and the role of lysyl
oxidase MICRO -staph aures food poisoning -fungal questions X3 -pork tape worm normal flora for nose -infant pnuemonia -hypersensitivity (for contact dermatiti
s) -Rhogam is PASSIVE immunity PATH -congenital heart defects -fragile X dz -ris
k of translocation with maternal age -Duchenne s MD is xlinked -CF: psuedomaonas
infections and autsomal recessive -marfans and dissecting aortic anuerysms -neu
ral tube defects and alpha FP -melanoma (depth of lesion is worst sign) -epidura
l hematoma (they called it extradural) -anemias -bleeding disorders -multiple my
eloma -obstructive lung dz -ARDS -arthritis -SLE -ankylosing spondylitis

-osteoporosis and compression fractures -hydatidiform mole is 46 XX -breast dz (


what a blocked lymph duct would look like??) -Coxsackie B (dilated cardiomyopath
y) -heart murmurs -temporal ateritis (tx: prednisone) -acid/base graphs -Xray of
pnuemothorax PHARM -maintanence dose -first order elim. -effect of antagonist i
n general -tetracycline mechanism (MORE than knowing 30S) -aminoglycosides=ototo
xicity -INH mechanism -cocaine additction mech. -PAM -Dantrolene mech. (Ca2+ fro
m SR) (yes, got it right Kelly) -Beta blockers -Alcohol and Benzodiazipenes -Sum
atriptan mech -Diuretics X5-6 -Quinidein efx X3 -Bleomycin side efx (cancer bear
will be your friend) -warfarin mech -misoprostol (w/NSAIDs) -H2 blockers -Arach
idonic acid pathway X3 -diabetic drugs (increasing insulin receptor sens) -predn
isone PHYS (not that many calculations, few were there weren t too bad) -cardic
output -cariac loop cycle (then inferring which one had largest CO) -control of
BP -filtration fraction -Glc. clearance -kidney hormones -PTH -uses of estrogen
-menopause profile -O2 diss. curve -secretion and bicarb -calculate calories (fr
om CHO, prot. and fat)

Step 1 Questions 2004 1- 57 y/o male is on hemodialysis for CRF. Which of the ff
g metabolic derangements can be anticipated? ABCDEhypercalcemia hypophosphatemia
osteomalacia Vit. D excess Hypoparathyroidism
Answer is c- osteomalacia. Kidneys failure to excrete phosphate hyperphosphatemia
& fail to synthesize 1,25(OH)2D3. then Vit. D def. impaired intestinal ca absorp
tion. Phosphate retention, defective intestinal absorption & skeletal resistance
to PTH hypocalcemia secondary hyperparathyroidism worsens hyperphosphatemia by
increasing the Ph. release from bone renal osteodystrophy
2- 50 y/o woman with a Hx of essential HTN, presents to ER with severe headache,
nausea, vomiting and photophobia. On P/E, BP= 160/100 mmHg, shes confused with n
uchal rigidity without focal neurologic signs. ** Whats the Dx? A- hemorrhagic st
roke B- ruptured cerebral aneurysm C- meningitis D- ischemic cerebrovascular acc
ident E- TIA
** Dx is best confirmed with? A- LP B- Doppler US of carotid artries C- Head MRI
D- Head CT followed by cerebral angiography E- EEG
** Whats the next most important step in Mx? A- urgent surgery with aneurysm clip
ping B- admission to ICU, close monitoring and Tx of HTN C- admission to ICU, cl
ose monitoring and IV antibiotics D- anticoagulation & antiplatlete therapy E- s
erial LP to drain CSF

Answers are B, D, A most likley the pt. ruptured cerebral anurysm, as pt. is con
fused, no focal signs present and pt. has nuchal rigidity, could be and most lik
ely due to the blood in the CSF--> meningeal irritation, etc.. although you also
see nuchal rigidity w/ meningitis, here you have confusion as well although the
y give us a nice high B.P of 160/100, and we might want to choose stroke over ce
rebral anurysm, given the rest of the prez. here anurysm is more likly, as well
as you would expect to see neuro focal findings w/ stroke, not present here isch
emic cerebrovascular accident --> expect to see some focal signs as well as may
not have nuchal rigidity TIA, usually transient and other symptoms present , not
seen here
3- During an elective laparascopic cholecystectomy, the patient developes a sudd
en onset drop in BP, atrial desaturation and an increase in ventilatory pressure
. Whats the most appropriate step of Mx? A- an IV fluid bolous B- decompression o
f the pneumoperitoneum C- inserting a chest tube D- re-evaluating the position o
f the endotracheal tube & portable CXR E- aborting the procedure & converting to
an open cholecystectomy Answer is C, coz patient has developed tension pneumoth
orax.
4- 2 wk old boy is brought for check up. Hes been doing well at home. Results of
his newborn screen shows he has sickle cell dis. Whats the most important step in
his Mx? A- avoiding heat exposure B- immunization with pneumococcal vaccine Cfolic acid supplements D- iron supplements E- prophylactic penicillin
** Six months later, the same patient returns with a 3 day Hx of lethargy & feve
r with rhinorrhea and cough. On P/E, he is pale, tacchycardic with a LUQ mass. H
gb= 4, Plt= 100000, WBC= 15000 with 50% segmented neut. , reticulocyte count=15%
. Whats the Dx?

ABCDEAcute chest syn. Acute splenic sequestration Aplastic crisis Intussusception Vas
o-occlusive crisis
Answer is E & B. because hes at high risk of infection, especially with encapsula
ted organisms and penicillin will dramatically decrease this risk. Pneumococcal
vaccine is not effective in neonates. Its usually given at 2 yrs age.
5- A 52 y/o man receives a preoperative evaluation before an alective surgery. H
e is asymptomatic with a normal P/E, but he is noted to have a Hgb= 10.8, Hct= 3
3, MCV= 70, RBC= 6.1x1000/ microlit. Whats the most likely Dx? A- Sickle cell dis
. B- Iron def. Anemia C- Alpha thalassemia major D- Beta thalassemia minor E- An
emia of chronic dis. Answer is D. low MCV is either IDA or beta-thal. BUT, high
no. of RBCs is characteristic of beta-thal. In IDA, RBC count is below normal. p
t. has hypocromic, microcytic anemia,..as well as increased retic. count--> incr
eased RBC s, ..this last feature makes the better choice here to be thalassemia,
as oposed to iron def. for example iron def. seems to be avery good choice, as
one should think anemia in older MAN , is due to cancer-COLON ca , until proven
otherwise..however you would expect to see low reticulocytes count w/ iron def.
anemia. note- elevated retic. counts--> seen in thalassemia/hemoglobinopathy,..l
ike SS anemia
6- match the clinical description with the most likely organism: abcdefghStrep.
Pneumoniae Staph. Aureus Strep. Viridans Providentia stuartii Actinomyces israel
ii Hemophilus ducreyi Neisseria meningitides Listeria monocytogene

1- 30 y/o female with MVP, MR develops fever, anorexia & weight loss after a den
tal procedure 2- 80 y/o male hospitalized for hip Fx, has foley cath. in place,
develops shaking chills, fever & hypotension. 3- young man develops painless, fl
uctuant, purplish lesion over mandible, after several weeks cutaneous fistula is
noted. 4- sickle cell patient presents with high fever, toxicity signs of pneum
onia & stiff neck.
Answers: 1- c, 2- d, 3- e, 4-a
7- 65 y/o male with Hx of DM & cardiomyopathy, presents with severe knee pain. O
n P/E, knee is swollen, red & tender. Knee X-Ray shows linear clcification. ***
Dx is best made by: A- Serum uric acid B- Serum calcium C- Arthrocentesis & iden
tification of birefringent rhomboid crystals D- Rheumatoid factor *** Further wo
rkup inthis patient should include evaluation for: A- Renal dis. B- Hemochromato
sis C- PUD D- Lyme dis. Answers are C & B. Acute monoarticular arthritis in asso
ciation with linear calcification in the cartilage of knee maqkes the Dx of pseu
dogout which is positive for birefringent crystals in joint fluid. Pseudogout ma
ybe associated with hemochromatosis.
8- 65 y/o woman with a 12 Hx of symmetric polyarthritis, presents with splenomeg
aly, ulcers on lat. Malleoli, synovitis of wrists, shoulders and knees, and no h
epatomegaly. Lab results : WBC=2500, RF= 1:4096, this patients WBC diff is most
likely to show what ? A- pancytopenia B- lymphopenia

C- granulocytopenia D- lymphocytosis E- basophilia Answer is C. case of feltys sy


n. ( RA+ splenomegaly+ leukopenia ), the mech. Of granulocytopenia is poorly und
erstood.
9- A patient with low grade fever & weight loss has poor excursion on the Rt. si
de of chest with decreased fremitus, flatness to percussion and decreased breath
sounds. Trachea is deviated to the Lt. Whats the most likely Dx? A- Pneumothorax
B- Pleural effusion secondary to histoplasmosis C- Consolidated pneumonia D- At
electasis Answer is B. Physical findings all consistent with pleural effusion, w
hich in large amount can shift trachea to the Lt. in pneumothorax hyperresonance
of the affected side is present. Atelectasis on the Rt would shift trachea to t
he Rt.
10- A patient have an unexpected high value for diffusing capacity, this finding
is most consistent with which of the ffg? A- Anemia B- Cystic fibrosis C- Emphy
sema D- Intrapulmonary hemorrhage Answer is D. Decreased diffusing capacity is s
een in: primary parenchymal disorder, anemia & removal of lung tissue. Increased
value is seen in : polycythemia, CHF & intrapulmonary hemorrhage.
11- A 65 y/o man is admitted to the hospital with anginal chest pain. His genera
l health has been excellent, although he has had a multinodular goiter for many
years. He had a series of thyroid function tests 4 weeks before admission, and t
he results are :
Serum T4 8.0 g/dL Free T4 index 8.0 Serum T3 152 ng/dL Serum thyrotropin (TSH) 0.
7 U/mL

A MI is ruled out, but chest pain continues. A coronary arteriogram shows a 90%
stenosis of the left main coronary artery. A coronary artery bypass graft is don
e. The patient has an uneventful postoperative course and is discharged on the s
eventh postoperative day. One month later, he is readmitted in atrial fibrillati
on with a rapid ventricular response. Repeat thyroid function testing shows the
following:
Serum T4 15.0 g/dL Free T4 index 15.8 Serum T3 220 ng/dL Serum TSH <0.01 mU/L
What is the most likely Dx? A. Graves disease B. Stress-induced hyperthyroidism
C. Iodine-induced hyperthyroidism D. Silent thyroiditis Answer is C. The natura
l history of multinodular goiters is slow growth and gradual decrease in thyrotr
opin (TSH), reflecting increasing thyroid hormone production. This progression o
ccurs over years to decades, however. Many patients with multinodular goiters ha
ve autonomous areas within their thyroid. This patient had normal thyroid functi
on 1 month before admission. However, his serum TSH level was near the lower lim
its of normal, suggesting the possibility of autonomous thyroid function. When p
atients with multinodular goiters are exposed to excess iodine, severe hyperthyr
oidism may occur. This is known as iodine-induced hyperthyroidism or the JodBase
dow phenomenon. When iodine supplementation is introduced into areas of iodine d
eficiency, iodine-induced hyperthyroidism may occur in patients with multinodula
r goiters. Iodine-induced hyperthyroidism may occur in nonendemic goiter areas a
s well, often with devastating consequences. The high iodine content of the dye
used for the cardiac catheterization undoubtedly precipitated the hyperthyroidis
m in this patient. The onset of hyperthyroidism may be delayed for several weeks
to months after the iodide exposure. Although other causes of hyperthyroidism a
re possible, none is as likely as this scenario. When patients with multinodular
goiter must be exposed to excess iodine (for example, during cardiac catheteriz
ation, computed tomographic [CT] scan with contrast medium, or amiodarone therap
y), premedication with antithyroid drugs (methimazole or propylthiouracil) shoul
d be considered.

12- A 43 y/o woman complains of itching that


examination is normal, except for the liver,
costal margin.The blood count is normal; the
e as follows:
Creatinine 0.8 mg/dL Bilirubin 0.6
4.2 g/dL Alkaline phosphatase 450 U/L

keeps her awake at night. Physical


which is felt 7 cm below the right
results of serum chemistry tests ar
mg/dL Alanine aminotransferase 78 U/L Albumin

Which test would you order next in order to diagnose the underlying disorder? (A
) Serum protein electrophoresis (B) Antismooth-muscle antibody (C) Antimitochond
rial antibody (D) Technetium-99m liver-spleen scan (E) ERCP
Answer is C. This is the classic description of primary biliary cirrhosis. Itchi
ng is the most common specific symptom of early primary biliary cirrhosis. Appro
ximately 70% of affected patients have enlarged livers. The best screening test
for suspected primary biliary cirrhosis is the antimitochondrial antibody test.
It is positive in 95% of affected patients and has a 98% specificity if newer en
zyme-linked immunosorbent assay (ELISA) tests are used. Serum protein electropho
resis might show a diffuse increase in immunoglobulins. However, this finding is
nonspecific and may be found in many chronic liver diseases. Antismooth-muscle
antibody tests are positive in some patients with autoimmune chronic hepatitis.
However, the test is nonspecific and not terribly useful. The sulfur colloid tec
hnetium liver-spleen scan is useful for detecting portal hypertension and hypers
plenism. However, it is not specific and would not be helpful in diagnosing prim
ary biliary cirrhosis. Endoscopic retrograde cholangiopancreatography (ERCP) is
typically normal in patients with primary biliary cirrhosis. Its only role in th
e diagnosis of primary biliary cirrhosis is in the patient who presents with a s
imilar syndrome but who has a negative antimitochondrial antibody test. ERCP wou
ld then be performed to look for other causes of disease such as primary scleros
ing cholangitis.
13- A patient presents to your office with complaints of severe earache and decr
eased hearing on the left side. Examination confirms left otitis media. You pres
cribe an oral cephalosporin and an analgesic. The next day the patient is brough
t to the emergency room with a rash and moderate bronchospasm. Reviewing his off
ice record you are horrified to discover that he is known to have a severe aller
gic reaction to penicillin.

What should you tell the patient at this point? (A) Apologize and explain what h
appened. (B) Apologize and offer to compensate him financially. (C) Attribute th
e rash and the bronchospasm to his infection. (D) Explain to the patient that he
is allergic to cephalosporin as well as penicillin. (E) No explanations are nec
essary. Answer is A. Medical errors are common but are infrequently reported to
patients. The fear is that acknowledging a mistake may lead to litigation. Never
theless, there is no excuse for a physician not to admit a mistake, which is an
inevitable part of medical practice. Apologizing and explaining is not only ethi
cal, but also has been shown to decrease the risk of litigation. There is no nee
d or obligation to offer financial compensation. Trying to attribute your mistak
e to other causes, or attempting to blame the patient for your negligence is not
only unethical, but will increase your risk of litigation. Finally, nothing wil
l anger a patient more than no explanation at all.
14- A 29 y/o woman presents with an exacerbation of her asthma. She is 11 weeks
pregnant. She has mild intermittent asthma and usually takes a -agonist as needed
. She has one 4-year-old child who is in day care and has had a recent upper res
piratory tract infection. She has a dry cough, clear nasal discharge, myalgias,
and fatigue. On physical examination, she is talking in full sentences and has n
ormal tympanic membranes, mildly erythematous oropharynx without exudates, no ad
enopathy; she has positive wheezing bilaterally. The peak flow is 300 mL; her us
ual result is 390 mL. Pulse oximetry is 93% on room air.Which of the following i
s indicated in the management of this patient? (A) Amoxicillin (B) Theophylline
(C) Prednisone (D) Montelukast (E) Flunisolide Answer is E. Treatment principles
for asthma in the nonpregnant patient apply also to the pregnant patient. She h
as mild intermittent asthma with an exacerbation, for which therapy with an inha
led glucocorticoid is an appropriate choice. Use of inhaled glucocorticoids is s
afe in pregnancy. Asthma in pregnancy has a variable course, with one third of p
atients getting better, one third getting worse, and one third staying the same.
This patient has had mild symptoms and now has an exacerbation related either t
o an upper respiratory tract infection, or to

worsening of her asthma because of her pregnancy. She has no signs of streptococ
cal pharyngitis, sinusitis, bacterial tracheobronchitis, or pneumonia; antibioti
cs should therefore not be given. Prednisone therapy should be reserved for furt
her symptoms. Theophylline can be used in pregnancy with close monitoring, but i
t is not the next drug of choice. Montelukast may be used in pregnancy, but shou
ld be reserved for progressive symptoms. The patient should continue therapy wit
h -agonists. Use of a peak flow meter at home is integral to her management; she
should be instructed and encouraged in its use.
15- A 30-year-old woman presents because of 6 months of amenorrhea. She had regu
lar menses starting at age 13 years and is otherwise healthy. Menses gradually b
ecame irregular over the past 5 years, in spite of relatively stable weight and
activity. She has no acne or hirsutism. The results of recent laboratory tests i
ncluded normal serum thyroidstimulating hormone and prolactin levels and a negat
ive serum pregnancy test. What is the next most useful diagnostic test for the e
valuation of secondary amenorrhea in this patient? (A) Measurement of LH (B) Mea
surement of FSH (C) Pelvic US (D) Measurement of serum estradiol (E) MRI of the
pituitary gland Answer is B. In this patient, the negative pregnancy test, and n
ormal serum TSH and prolactin levels exclude the most common causes of secondary
amenorrhea. The remaining causes include hypothalamic amenorrhea and ovarian fa
ilure. Of the tests listed, serum FSH is the most likely to be diagnostic of a r
eproductive endocrine abnormality, as it has a longer half life than luteinizing
hormone and becomes elevated relatively early in the process of ovarian aging.
Luteinizing hormone is less useful because of its greater variability across the
normal menstrual cycle (including the dramatic, often 10-fold, increase at the
mid-cycle surge), its more striking 1 to 2 hourly pulsatility, and its elevation
in polycystic ovary syndrome as well as ovarian failure. Serum estradiol is fre
quently in the normal range in various reproductive disorders and therefore rare
ly useful. MRI of the pituitary gland is not indicated unless other tests are no
rmal and a hypothalamic cause of amenorrhea needs to be ruled out. Finally, pelv
ic ultrasound can show the presence of ovaries, their size, and the uterine morp
hology and endometrial thickness, but cannot diagnose the cause of secondary ame
norrhea. It may be more useful in the evaluation of primary amenorrhea, if the p
hysical examination is inadequate to confirm the presence of intact ovaries and
a uterus, or in the evaluation of excessive menstrual bleeding, when it can iden
tify endometrial thickening, polyps, and/or

uterine leiomyomas. A medroxyprogesterone challenge test can show that the uteri
ne anatomy and vaginal outflow track are normal, which is suggested by the histo
ry of previously normal menstrual cycles. In addition, the challenge test provid
es some information about recent estrogen exposure. However, it cannot be diagno
stic in estrogen deficiency.
16- A 2 y/o has a chronic cough. CXR reveals hyperinflation of the left hemithor
ax. Bilateral decubitus views showed that the right lung becomes appropriately a
telectatic however, the left side shows no change in appearance with decubitus p
ositioning. Whats the most likely diagnosis? Asymmetric hyperinflation suggests t
he possibility of an aspirated foreign body in this two year old infant. These f
indings suggest the diagnosis of an aspirated foreign body or a possible obstruc
tion of the left main stem bronchus from a central etiology. Bronchoscopy is bot
h diagnostic and therapeutic.
17- match this drugs with their associated syndrome: ABCDEFbarbiturates Ecstasy
Inhalants Marijuana Methamphetamine PCP
123456Severe encephalopathy Lung cancer Rhabdomyolysis during intoxication Wanting to
touch/be touched during intoxication Seizures during withdrawal Swiss cheese app
earance on functioasdfasdfasdfdghsdfgsdf. Chronic bronchitis Exacerbation of ast
hma Pneumonia
Answer is A- Acute exacerbation of COPD, it occurs when patient develops acute o
nset of marked dyspnea & tachypnea with use of accessory mucles with no response
to medication.

20- 73 y/o male with Hx of HTN, presents with short episode of Lt. sided weaknes
s & slurred speech. Also he has a Hx of 3 brief episodes of sudden Rt. eye visio
n impairment in the last month. 1- Whats the best next diagnostic test? ABCDECere
bral MRI Holter monitoring Visual evoked responses Carotid artry doppler US Conv
entional cerebral angiography
2- Episodes of visual loss are related to: ABCDERetinal vein thrombosis Central
retinal A. ischemia Post. cerebral A. ischemia Middle cerebral A. ischemia Post.
ciliary A. ischemia
Answers are D & B. This is a classicd case of extracranial internal carotid A. d
is. which include episodes of ipsilat. transient monocular blindness ( amaurosis
fugax ) & contralat. TIA consisting of motor weekness. The most appropriate tes
t to confirm the Dx of carotid stenosis is Doppler us. The mech. of transient mo
nocular blindness is embolism to the central retinal A. or one of its branches.
21- A 5 y/o presents to the ER 3 h after a possible button battery ingestion. Th
e patient is in no acute distress, vital signs are stable, and examination is be
nign. CXRshows what appears to be a small button battery in the stomach. Which o
f the following is the MOST appropriate next action? (A) Upper GI series to furt
her delineate the exact location of the foreign body (B) Attempt battery removal
by the Foley balloon catheter technique (C) Immediate GI consultation for endos
copic removal (D) Immediate surgical consultation (E) Discharge to home with par
ental observation and weekly radiographs The answer is E. Button batteries lodge
d in the esophagus require emergencyremoval to avoid esophageal burns and perfor
ation. If the button battery has passed the esophagus and the patient is asympto
matic, home observation with serial x-rays toensure passage through the pylorus
is the appropriate course of action. Most button batteriesthat have passed the e
sophagus will transit through the entire body within 24 to 48 h without difficul
ty. If the battery is of large diameter and the patient is younger than 6

years, the battery is less likely to pass, and endoscopic retrieval is the prefe
rred treatment.
22- A 45 yo male with a long Hx of alcohol use and presumptive gastritis present
s to the with sudden onset of severe abdominal pain and vomiting. V/S: BP=110/60
, HR=110/min , T=101F, RR= 30/minhes diaphoretic with epigastric tenderness and mi
ld guarding. Lab data: WBC= 30,000, amylase =2,000. CXR shows a small amount of
free air under the diaphragm. What is the MOST likely diagnosis? (A) Acute pancr
eatitis with associated Mallory- Weiss syndrome (B) Acute pancreatitis with asso
ciated Boerhaaves syndrome (C) Acute pancreatitis with associated enzymatic destr
uction of bowel wall (D) Acute pancreatitis secondary to anterior duodenal ulcer
perforation (E) Acute pancreatitis secondary to posterior duodenal ulcer perfor
ation answer is E. Because the pancreas adheres to the posterior duodenum, ruptu
red posterior duodenal ulcers generally penetrate into the pancreas rather than
perforate into the free peritoneum. Anterior ulcers are more likely to perforate
into the peritoneal cavity. A Mallory-Weiss tear of the esophageal wall usually
presents with symptoms similar to reflux esophagitis and causes moderate, selflimited bleeding. Patients with Boerhaaves syndrome rapidly deteriorate to a stat
e of shock and septicemia due to a malignant mediastinitis.
23- A 55 y/o male without significant medical history presents with LLQ pain and
constipation. V/S: T= 100.5F, PR= 85, BP=150/80, RR=12. P/E is unremarkable exce
pt for mild LLQ tenderness without guarding and rebound. Rectal examination show
s hemenegative stool and no tenderness.Lab results are within normal limits exce
pt for WBC=13,000 with a left shift. Which of the following would be the MOST ap
propriate management for this patient? (A) Prompt surgical evaluation in the ED
(B) Emergent upper GI series (C) Emergent barium enema (D) Discharge to home wit
h bowel rest and oral antibiotics (E) Discharge home with repeat abdominal exami
nation in 12 h or sooner if worse The answer is D. The patient described in the
scenario most likely has diverticulitis. Patients with localized pain and no sig
ns and symptoms of peritonitis or systemic infection do not require hospitalizat
ion. Outpatient management consists of bowel rest, broad-spectrum oral antibioti
c therapy, and close follow-up.
24- Which of the following etiologic agents is the MOST common cause of infectio
n in liver transplant patients? (A) Candida (B) Cytomegalovirus (C) Herpes simpl
ex virus

(D) P. carinii (E) L. monocytogenes The answer is B. Complications of infections


in liver transplant patients account for nearly 90 percent of deaths. The most
common infectious agent after transplantation is Cytomegalovirus (CMV. Reported
occurrence ranges from 23 to 85 percent of all liver transplant patients. Fortun
ately, CMV is rarely fatal.
25- A 56 y/o heavy alcohol user male presents with vomiting blood for several ho
urs. BP=90/60, PR=110, RR=16, and T=98F. Placement of an NGT shows active bright
red bleeding. Which of the following is the treatment of choice? (A) Tamponade w
ith a Sengstaken-Blakemore tube (B) Therapeutic upper GI endoscopy (C) Octreotid
e infusion (D) Vasopressin infusion (E) Immediate referral for surgical interven
tion The answer is B. GI bleeding is a common problem seen in the ED and is pote
ntially life threatening. For patients with significant active bleeding, emergen
cy endoscopy is the treatment of choice. Esophageal varices can be treated endos
copically with either band ligation or sclerotherapy, resulting in control of ac
ute bleeding in up to 90 percent of patients. Hemostasis can be achieved with no
nvariceal sources of bleeding as well. Drug therapy with both octreotide and som
atostatin reduces bleeding from both varices and PUD and is a useful adjunct to
endoscopy. Vasopressin therapy has largely been discontinued due to a high rate
of adverse effects including hypertension, arrhythmias, myocardial ischemia, and
decreased cardiac output. Balloon tamponade can be used as a temporizing measur
e to control bleeding by placing direct pressure on the gastric and esophageal m
ucosa. However, it is frequently associated with complications, including mucosa
l ulceration, esophageal or gastric rupture, asphyxiation from dislodged balloon
s, and aspiration pneumonia. Although it is appropriate to make a surgeon aware
of a critical patient, emergency surgical intervention is indicated only in thos
e patients who fail endoscopic hemostasis and medical therapy.
*** 36 y/o woman with meno/metrorrhagia, inlarged uterus, which of the ffg would
tell the severity of this condition? a- CBC b- Hysteroscopy c- US d- Pelvic exa
m

Its US, when severity of the condn is asked, it need not necessarily mean bleedin
g (submucosal and intramural) it could be pressure sym also(subserosal), hence U
S. to add, since 36 yr most common is fibroids since adenomyosis in the premenop
ausal, tho both can have meno metrorrhagia.
*** whats the most effective way to prevent kidney stones? a- exercise b- diet chydration d- periodic U/A *** 25 y/o female G1P0, 16 wks of gestation, presents
with vaginal bleeding & cramps. No products of conception is expelled yet. Whats
the next step of Mx? a- Adimt to hospital, observation & monitoring b- Bed rest
at home c- Admit to hospital, D&C *** Which of the ffg is a contraindication fo
r vaginal delivery after previous c/s ? a- Low segment transverse uterine incisi
on b- breech c- clinically adequate pelvis d- placenta previa a. repeat cs possi
ble, unless longitudinal incision, not done nowadays. b. vaginal can be done. c.
prerequisit for vaginal. d. regardless of previous delivery, since the incision
would be on the low implanted placenta- torrential bleeding unless the (lower p
lacental edge is more than 2cm from the internal os and it the first delivery wi
thout previous CS answer is d. *** 23 y/o female with primary amenorrhea, on P/E
breats are abesnt, but uterus is present, what test will u orderfirst ? a- FSH
b- Karyotype c- Testosterone d- estrogen FSH. Good explanation in first aid..

*** Post MI pulmonary edema, what do you do next? reduction of pulmonary venous
return (preload reduction) and reduction of systemic vascular resistance (afterl
oad reduction) with diuretics ( furesmide ) & morphine sulfate, in a case of hyp
otension give inotropic agents, supplemental O2.
*** 50 y/o male with known Hx of TB, presents with shoertness of breath, CXR sho
ws massive Rt. side pleural effusion, whats the next step? a- CT b- Thoracocentes
is c- Pericardiectomy d- etc. b- thoracocentesis
*** in coarctation of aorta: 1- peripheral resistance a- increase b-decrease 2pulmonary resistance a- increase b- decrease 1- decrease, 2- increase
*** to prevent hepatitis B in a IV drug abuser, what would you do? a- HBV vaccin
e b- HBV Ig c- Both d- etc
*** patient presents with trauma to base of neck, damage to which structure caus
es most severe compromise of cardiorespiratory system? a- thoracic aorta b- jugu
lar vein c- esophagus d- SVC
19 y/o with a small lump in her Lt./ breast is very concerned that its malignant
cancer, work up & Bx shows its benign, but she is still extremely worried inspite
of reassurance by her physician. Which of the ffg is the best Tx in this case?
A- a careful explanation of benign nature of the complaint B- Use of benzodiazep
ine C- Skillful physician reassurance D- Use of placebo medication

E- Psychotherapy to explore her current life circumstances Answer is E. case of


hypochondriasis which is usually becomes evident during psychological stress.
Which of the ffg risk factors are related to breast carcinoma? A- Obesity, late
manopause, first pregnancy > 35 yr, smoking, high fat diet B- Obesity, late meno
pause, low fiber diet, smoking C- First pregnancy>35 yr, obesity, smoking, famil
y Hx, age D- Low fiber diet, age, smoking, first pregnancy> 35 yr, late menopaus
e E- Age, obesity, late menopause, first pregnancy>35 yr, low fiber, high fat di
et Answer is E. Smoking is not a risk factor. The most important risk factors ar
e: - Age - Family Hx - Nulliparous state - Early menarche - Late menopause - Hx
of contralat. breast cancer - First pregnancy > 35 yr - High fat diet - Low fibe
r diet
2 y/o boy who palys in a sandbox in a nursery that has couple cats for children
to play with, presents with wheezing, hepatosplenomegaly & peripheral blood eosi
nophilia. Whats the Dx? A- Pinworm infestations B- Lofflers syn. C- Ascariasis DVisceral larva migrans E- Strongyloidiasis Answer is D. Visceral larva migrans i
s caused by Toxocara larva. Its most common in children 1-4 y/o especially who ha
ve close contacts with digs & cats. Sandboxea are common places for both pets &
children. Sx are: fever, hepatosplenomegaly, wheezing, pulmonaer dis. & eosinoph
ilia
17 y/o woman G1P0 with 10 yr Hx of asthma undergoes pulmonary evaluation & spiro
metery. She doesnt have any respiratory complaints or Sx. Which of the ffg regard
ing her status during pregnancy is true? A- RR increases

BCDEVital capacity decreases Minute ventilation increases Functional residual capaci


ty remains unchanged Tidal volume decreased
Answer is C. since Px doest have any respiratory problem, shell undergo the normal
changes in respiratory physiology that occur during pregnancy: - RR is unchange
d - TV is increased - Minute ventilation is increased - VC remains unchanged - F
RC is decreased ( uterus elevates the resting position of diaphragm )
35 y/o woman with major depressive disorder, single episode has responded well t
o imipramine after 1 month of Tx. Which of the ffg is the most appropriate next
step? A- Continue imipramine for 6 months B- Continue imipramine indefenitly CGradual imipramine decrease till shes medication free, unless depression occurs D
- Stop imipramine immediately E- Switch to fluoxetine Answer is A. Maintanace th
erapy after response to antidepressants should be continued for 6 months after i
nitial response.
14 y/o girl presents with trouble concentrating at school just three months afte
r witnessing her best friend being shot to death by another classmate. Ever sinc
e that "awful day" she has been very "sad" and withdrawn, often sitting on her b
ed, staring at the wall "for hours." She has frequent crying spells and refuses
to play with friends or participate in her normal after-school activities. Her a
ppetite has "dwindled down to nothing" and she feels very guilty that she surviv
ed the shooting. On further questioning, she reluctantly admits that she has con
stant thoughts of "joining her friend." She did not receive any counseling after
the incident. *** Whats the most likely Dx? A. adjustment disorder B. brief psyc
hotic disorder C. dysthymic disorder D. major depressive disorder E. normal grie
f F. PTSD

*** Whats the most important q to ask at this time? A. "Are you hearing voices?"
B. "Do you have any friends that you feel comfortable talking to at school?" C.
"Do you wear a helmet when you ride your bicycle?" D. "Can you describe your typ
ical weekly alcohol intake?" E. "Have you thought of the means by which you can
join your friend ?"
1- The correct answer is D. This patient most likely has major depressive disord
er. To meet the criteria for this disorder a patient must exhibit a 2-week histo
ry of a distinct change in mood or a loss of interest or pleasure, along with at
least 4 of the following: a decreased appetite and weight loss, difficulty slee
ping, psychomotor retardation or agitation, fatigue, feelings of worthlessness o
r guilt, an inability to concentrate, and suicidal ideation. The symptoms must c
ause functional impairment. ( Kaplan, step 3 samples ) 2- The correct answer is
E. Since she has already told you that she has suicidal ideation ("joining her f
riend"), it is very important to ask if she has "thought of the means by which"
she can "join her friend" or has made any preparatory actions.
A 64 y/o married man has been diagnosed to be HIV positive. He comes in to the o
ffice to discuss the results and begs you not to tell his wife, who is also your
patient. He says that hearing what he has been in involved in "will kill her."
You remind him that HIV may also "kill her." The most appropriate first step is
to: A. contact his wife anonymously and tell her that she has been exposed to HI
V B. contact the appropriate government health agency and report your findings C
. promise him that as his doctor you will respect his privacy and maintain confi
dentiality D. promise him that you will keep the results confidential if he agre
es to use condoms with his wife E. try to persuade him to voluntarily discuss th
e issue with his wife The correct answer is E. Physicians must violate confident
iality and warn third persons about the danger of HIV infection if the patient i
s unwilling to inform the person himself. Before informing third parties, the ph
ysician should try to do everything possible to persuade the patient to voluntar
ily discuss the issue with their partner. If the physician believes that an indi
vidual may really be saved from a deadly infection, confidentiality should be vi
olated.

A 4 y/o boy presents with fever, irritability, and erythema of the hands and fee
t for the past week. His mother has been giving him aspirin to reduce his temper
ature. P/E on admission showed a T=39.7 C (103.4 F), bilateral conjunctival inje
ction, an enlarged right-sided cervical lymph node (1.8-cm), fissured lips, a re
d tongue with red papillae, pharyngeal hyperemia, erythematous and edematous pal
ms and soles, and a confluent, blanching erythematous rash on the trunk. IV flui
ds were started, the aspirin therapy was continued. Laboratory studies show ESR=
28mm/h Plt= 490,000/mm3. The patient is extremely uncomfortable and now shows d
esquamation of the fingers and toes. The most appropriate therapy at this time i
s: A. corticosteroids B. ibuprofen C. intravenous gammaglobulin D. oxacillin E.
penicillin V The correct answer is C. This patient most likely has Kawasaki dise
ase, which is treated with aspirin and intravenous gammaglobulin. The disease is
characterized by a high fever for longer than 5 days, bilateral conjunctival in
jection, fissured lips, a "strawberry tongue", mucosal change in the oral pharyn
x, erythematous and edematous palms and soles with desquamation, a polymorphous
rash, cervical lymphadenopathy, an elevated erythrocyte sedimentation rate, and
thrombocytosis. The most important complication is coronary artery aneurysms, wh
ich may be prevented by early treatment with aspirin and intravenous gammaglobul
in. An echocardiogram is necessary to evaluate cardiac involvement.
47 y/o woman with a Hx of similar attacks of epigastric abdominal pain in the pa
st was admitted to the hospital with a Dx of gallstone pancreatitis. She was NPO
and IV fluid started. On the evening of admission day, the patient is noted to
have T=103.4 F. Her BP & HR are within normal range. Her abdomen is diffusely te
nder to palpation with guarding. Whats the most appropriate management at this ti
me? A. draw blood cultures and await results B. draw blood cultures and initiate
ampicillin, gentamicin, and metronidazole therapy C. draw blood, urine, and spu
tum cultures and await results D. obtain an urgent abdominal CT scan E. start am
picillin, gentamicin, and metronidazole therapy The correct answer is B. The mos
t appropriate management at this time is to draw blood cultures and initiate amp
icillin, gentamicin, and metronidazole therapy. Intravenous antibiotics are only
indicated if there is evidence of pancreatic necrosis or if the patient develop
s a fever after the diagnosis of pancreatitis is made. There is a substantial am
ount of clinical literature validating this approach to treating pancreatitis. T
he appropriate sequence of events is to draw blood cultures prior to initiating
therapy in order to

maximize chances of detecting an organism.


A 24 hour old male infant is noted to have some peculiar jerking movements of th
e right foot and arm. His axillary temperature an hour before was 36.2"C. The ph
ysical examination reveals no unusual findings except that he appears small and
premature. His birth weight was 2,550 g. The mother s and infant s history show
that he was the second of twins born after 37 weeks gestation, presented in tr
ansverse position and his heart rate had dropped to 80 per minute 10 minutes pri
or to birth, with documented fetal hypoxia. He had an Apgar score of 3 at 1 minu
te and 7 at 5 minutes. The mother had nausea and vomiting during pregnancy for w
hich she was given vitamin B6. She had mild preeclampsia at delivery. *** Which
is the most likely diagnosis? a) b) c) d) e) Brain tumor Hypoxemia in utero and
possibly during delivery Cerebral trauma during delivery Vitamin B6 dependency N
one of the above
*** In the diagnostic work-up, you may obtain the following tests or procedures,
EXCEPT: a) b) c) d) e) CT scan of the head Lumbar puncture for examination and
culture of spinal fluid Electroencephalography Serology for toxoplasmosis Blood
levels of sugar and calcium
B&D Hypoxemia is the leading cause of seizures considering the history of drop i
n fetal heart rate and low Apgar score at 1 minute. Hypoglycemia, hypocalcemia,
vitamin B6 dependency, and meningitis are possible causes of seizures but unlike
ly and should be ruled out. Some cerebral anomaly is possible. Incidence of cong
enital malformations is higher in twins than , in singletons, and central nervou
s system malformations lead all others in frequency. Brain tumors at this age ar
e rare and usually present with recurrent vomiting and not with seizures.

An infant who was healthy at birth is brought to your office for her first offic
e visit at the age of 6 weeks. You notice that the infant is jaundiced and that
there is bilirubin staining of the wet diaper. Which one of the following diagno
ses is most consistent with these findings? a. Physiologic jaundice of the newbo
rn b. Hemolysis secondary to Rh incompatibility c. Crigler-Najjar syndrome d. Gi
lbert s syndrome e. Biliary atresia Answer is e- biliary atresia. bilirubbin in
urine a clue of direct hyperbilirubinemia
A 22 y/o man used illicit IV drugs briefly 1 year ago. He is asymptomatic and ha
s no history of viral hepatitis. Serum ALT and bilirubin concentrations are norm
al. Serologic studies show: Hepatitis B surface antigen (HBsAg) Hepatitis B core
antibody (HBcAb) Hepatitis B e antigen (HBeAg) Antibody to surface antigen (Ant
i-HBs) Antibody to e antigen (Anti-HBe) positive positive negative negative posi
tive
Which of the following statements best describes the current condition? a-He is
in the incubation period, is highly infectious, is likely to develop acute hepat
itis B. b-He is in the incubation period, is minimally infetious, and is likely
to develop acute hepatitis B. c-He is a chronic carrier of hepatitis B and is hi
ghly infectious. d-He was infected previously with hepatitis B. has recovered an
d is immune to hepatitis B. Answer is a.
A 35 y/o woman requests a routine health assessment. She has no specific medical
complaints or past medical hstory. Her father developed colon cancer at age 54
but is still living. His brother died of colon cancer at aae 41. Their father (t
he patient s grandfather)

died in his sixties of colon cancer. His sister died of gastric cancer. The pate
nt s sister was recently diagnosed with endometrial cancer at age 43. The patien
t has a good appetite and has no nausea, vomiting, or weight loss. She has had n
o change in her bowel habits or hematochezia, and her menstrual periods are norm
al. Physical examination is normal. Digital rectal examination is negative for f
ecal occult blood. A screening complete blood count and biochemical profile are
normal. Which of the following actions is most appropriate? a. Perform annual oc
cult blood tests of three stool specimens. Initiate screening with flexible sigm
oidoscopy at age 50. b. Perform colonoscopy at least every 2 years. c. Barium en
ema now; if normal no further evaluation. d. Perform colonoscopy now; if no poly
ps are seen, no further investigation is necessary. e. Perform fecal occult bloo
d testing and flexible sigmoidoscopy now. If no polyps are seen now, the patient
should be followed routinely with fecal occult blood testing and flexible sigmo
idoscopy beginning at age 50. Answer is b.
A 20 y/o woman is evaluated for fever of nine weeks duration. During this time sh
e has had daily temperature elevations to 40.0 C (104.0 F). The only other sympt
oms have been malaise and occasional aches in the hands and knees; on one occasi
on she noted a transient pink rash on the abdomen. A one-week course of ampicill
in had little effect. T=38.9 C (102.0 F); PR=108 per minute, and rhythm is regul
ar. A grade 1/6 systolic ejection murmur is heard best at the LSB. The spleen is
palpable 3 cm below the left costal margin on deep inspiration. Electrocardiogr
am and chest radiograph are normal. Laboratory studies: Hct Hgb WBC 35% 11.5 g/d
L 12,800/cu mm; 81%neutrophils, 4% monocytes, 14 lymphocytes, 1% eosinophils 250
Todd units {<200} Negative Negative Pending
ASO RF ANA Blood cultures

Which of the following is the most likely diagnosis? (A) (B) (C) (D) (E) Answer
is e. Enteric fever Bacterial endocarditis Lyme disease Hodgkin s disease Adultonset Still s disease
A
d
e
?

healthy 71 y/o man describes visual loss in his right eye. Flashes of light an
a curtainlike loss of lateral vision began when he awoke eight hours ago. Thes
symptoms have persisted. Which of the following is the most likely explanation
(A) Retinal vein occlusion (B) Retinal detachment Atheroembolic occlusion of a
lateral branch of the right retinal (C) artery (D) Ocular migraine (E) Occipita
l lobe seizure Answer is B.
What is the marker for CREST syn. ? A. anti-scl-70 B. anti centromere C. anti ds
-DNA D. SS-A (Ro) The marker for CREST is anti centromere. They both have C in i
t. anti-scl-70 is for scleroderma diffuse type. They both have scl in it. D. ant
i ds-DNA is for SLE. E. SS-A is for Sjogren s disease
Whats the characteristics of rapidly progressive glomerulonephritis (RPGN)?

A. crescent formation B. "lumpy-bumpy" subepithelial deposits C. IgM mesangial d


eposition D. Associated with Hep C the answer is A. Rapidly progressive glomerul
onephritis is characteristic of rapidly losing at least 50% of its glomeruli in
as short period of time--usually days to 3 month max. Extensive fibrinoid necros
is is found on biopsy.
Pt. In mVA was brought to ER, you suspected cardiac tamponade what is next step?
a. pericardiocentesis b. echo first.. C. CXR d. CT Answer: depending on senario
if pt is unstable proceed to Pericardiocentesis, If stable echo first.
A patient has end stage pancreatic cancer and as her primary care pysician, she
asks you how long she could survive. What you should tell her. a. 5 year rate is
30% b. 5 year rate is 20% c. " 15% d. " < 5% Answer: D- The overall 5-year surv
ival rate for this disease is less than 5%.
A 24-year-old white primigravida has developed several 1- to 2-mm erythematous p
apules on her abdomen in the third trimester. They are pruritic and tend to appe
ar in her striae. Liver function tests and a CBC are normal. Which one of the fo
llowing is the most likely diagnosis? a. Pruritus gravidarum b. Spangler s papul
ar dermatitis c. Impetigo herpetiformis d. Herpes gestationis e. Pruritic urtica
rial papules and plaques (PUPP) The ans is e The findings in this patient are mo
st consistent with PUPP. This condition is usually benign, is not associated wit
h increased fetal morbidity, and resolves after delivery, and

there is usually no recurrence in subsequent pregnancies. Herpes gestationis, im


petigo herpetiformis, and Spangler s papular dermatitis have different presentat
ions and may be associated with increased fetal morbidity. Pruritus gravidarum i
s characterized by pruritus without skin lesions.
Ultrasonography reveals placenta previa in a 41-year-old asymptomatic G4P3 at 21
weeks gestation. Appropriate management would be a. weekly speculum examination
s under aseptic conditions beginning in her third trimester to assess the risk o
f bleeding b. an MRI scan, with a repeat scan later in the pregnancy if indicate
d c. repeat ultrasonography in her third trimester d. cesarean delivery at 28 we
eks gestation if her L/S ratio is favorable e. reassurance that ultrasound diagn
osis of placenta previa without evidence of bleeding is no cause for concern and
can be disregarded The ans is c The incidence of placenta previa ranges from 6%
to 45% in the second trimester, but more than 95% of these resolve by the third
trimester. However, it remains a cause for concern and should be watched, not i
gnored, even if there is no bleeding. This patient should have repeat ultrasonog
raphy in her third trimester. An MRI is very helpful but need not be used except
in a difficult diagnostic situation. Weekly speculum examinations would create
a risk of hemorrhage. Delivery at 28 weeks would not be appropriate in a patient
with no symptoms and without confirmation of the persistence of placenta previa
.
Which one of the following is an absolute contraindication to tocolytic treatmen
t for preterm labor? a. Urinary tract infection b. Documented gestation less tha
n 28 weeks c. Chorioamnionitis d. Uncontrolled diabetes mellitus e. Any vaginal
bleeding due to mild abruptio placentae The ans is c Before tocolytic treatment
is instituted, absolute contraindications to tocolysis must be ruled out. These
include chorioamnionitis, severe abruptio placentae, severe bleeding from any ca
use, severe pregnancy-induced hypertension, fetal death, fetal anomaly incompati
ble with life, and severe fetal growth retardation. Chorioamnionitis may precipi
tate preterm labor and is an absolute contraindication to tocolysis. It may be p
resent in a febrile pregnant patient even with intact membranes. In this case

amniocentesis may be required to rule out infection. There are also a number of
relative contraindications. These include uncontrolled diabetes, hyperthyroidism
, maternal cardiac disease, mild chronic hypertension, mild abruptio placentae,
stable placenta previa, fetal distress, fetal anomaly, mild fetal growth retarda
tion, and cervical dilatation greater than 5 cm. In patients with relative contr
aindications to tocolysis the risk of complications from prematurity must be wei
ghed against the risk of tocolysis. Not all vaginal bleeding is due to a serious
obstetric condition. Cervical effacement or dilatation may be the cause. Even i
f the source of bleeding is determined to be a placental abruption, if the bleed
ing is minor, the abruption is mild, and the fetus is not in distress, tocolysis
is not absolutely contraindicated. While diabetes mellitus may be adversely aff
ected by beta-adrenergic tocolytic agents, it is not an absolute contraindicatio
n to tocolysis. Close glycemic monitoring is, of course, mandatory. Even a few w
eeks of effective tocolysis may significantly alter the perinatal outcome of ges
tations between 25 and 27 weeks. Gestational age less than 28 weeks is therefore
not a contraindication to tocolysis. While a urinary tract infection may precip
itate preterm labor, tocolysis is not contraindicated. The infection, of course,
should be treated.
Which one of the following statements is most accurate concerning juvenile rheum
atoid arthritis? a. Fever is a rare systemic manifestation b. Ten years after th
e onset of disease, most patients have excellent functional status c. Most patie
nts have a permanent deformity of at least 1 extremity d. The disease is charact
erized by lifelong recurrences e. Most patients require corticosteroid treatment
answer is b. At least 50% of patients followed for up to 15 years have complete
remission of juvenile rheumatoid arthritis, and 70% regain normal function. A f
ew patients are left with crippling joint deformities, but 75% have no significa
nt residual deformity. Systemiconset disease is accompanied by high fever, rheum
atoid rash, polyarthritis, and other systemic manifestations
A 5-year-old African-American male fell off his bicycle and hit the back of his
head on a hard surface. There was no loss of consciousness. No other injury was
noted. He was obviously agitated and restless, and his only complaint was a loss
of vision. When you see him, his examination is unremarkable except for moderat
e swelling over the occipitoparietal area of the scalp. His skin is intact, and
no gross neurologic deficit is noted except for the visual loss. A CT scan is ne
gative. An EEG shows only slight slowing of activity. Which one of the following
statements is true regarding this patient?

a. The child s vision will probably return within 24 hours b. The child should b
e hospitalized for 72 hours for further observation c. It will be months before
the child s vision returns d. The loss of vision is probably caused by damage to
the optic nerve Ans is a Transient cortical blindness following mild head traum
a is usually associated with a benign outcome. The special features are mild hea
d trauma, no loss of consciousness, onset of blindness occurring within hours of
the trauma, duration of blindness less than 24 hours, absence of skull fracture
or visible injury on CT scan, and no other neurologic deficits. The EEG shows i
nitial slowing with normalization on follow-up.
which of the following medication is known to exacerbate psoriaisis? a)prednison
e B NSAID c hydroxychloroquine d methotrexate answer is c.
1- A 20 y/o man who moved to the US from Cambodia 2 years ago presents to the ER
with the sudden onset of left hemiparesis. His wife, who emigrated with him, re
ports that he has not felt well for at least a month and has had a weight loss o
f about 9 kg (20 lb). On P/E, T=38 C (100.4 F) and BP=116/52 mm Hg. He is somewhat
cachectic and hemiparetic. The cardiac examination shows a murmur. The physical
examination is otherwise normal.Which of the following tests is most likely to
give the diagnosis? (A) Blood cultures (B) Lumbar puncture (C) Radiography of th
e chest (D) Complete blood count, differential, platelet count, and erythrocyte
sedimentation rate (E) Biopsy of inguinal lymph node (F) head CT scan Answer is
A.The appearance of focal neurologic signs in a young person from Cambodia raise
s a wide differential diagnosis. Atherosclerotic cerebrovascular disease is unco
mmon in the 20-year-old age group. Among other diagnostic considerations are an
embolic event from a cardiac source, such as a valvular vegetation associated wi
th endocarditis or

an atrial myxoma; vasculitis; tuberculomatous or bacterial brain abscess; brain


tumor; aneurysm or arteriovenous malformation; and coagulation disorder such as
thrombotic thrombocytopenic purpura or hyperviscosity syndrome caused by multipl
e myeloma. Neurologic complications occur in 25% to 40% of patients with infecti
ve endocarditis. About 15% develop cerebral emboli with associated neurologic sy
mptoms. The easily associated triad of new focal neurologic deficits, fever, and
changing heart murmur occurs in only 33% of patients, so infective endocarditis
must be considered in any patient with sudden focal neurologic deficits who has
no conventional risk factors, such as atherosclerosis.
2- A 35 y/o woman presents with a 1.5-cm Lt breast mass which is nodular with in
distinct borders. Her mother and maternal aunt both had breast cancer in their f
orties. How would you evaluate this woman s condition? (A) Observe through a men
strual cycle (B) Diagnostic mammogram (C) Breast ultrasound (D) Office needle as
piration This woman has a significant risk for breast cancer. The suspicion of c
ancer is high because of the characteristics of the mass and the family history
of early-age breast cancer. The first approach would be a diagnostic mammogram t
o further define the lesion and, equally important, to look for suspicious lesio
ns in the opposite breast. Even if the mammogram is negative, referral to a surg
eon for biopsy would be the next step.
3- A 22-year-old female primagravida is seen in prenatal clinic 5 months after h
er last menstrual period. Her pregnancy has been uneventful, and she has gained
weight progressively over the last 2 months. Three months ago, her blood pressur
e was 120/80 mm Hg, and there was 2+ protein on dipstick urinalysis. Today she h
as a blood pressure of 150/95 mm Hg and marked bilateral lower extremity edema.
Laboratory studies: Hematocrit Leukocyte count Platelet count 29.7% 4200/L 209,0
00/L

Blood urea nitrogen Serum creatinine Serum uric acid Urinalysis: Protein Microsc
opic 24-hour urine protein excretion Creatinine clearance
15 mg/dL 1.1 mg/dL 6.0 mg/dL 4+ Hematuria, rare erythrocyte casts, rare broad ca
sts, few leukocytes 12.5 g 80 mL/min
This clinical presentation is most consistent with: (A) Preeclampsia (B) Underly
ing renal disease present before conception (C) Hypertensive nephrosclerosis (D)
Pyelonephritis Answer is B. In this 22-year-old pregnant woman, it is critical
to determine whether previously existing underlying renal disease is present bec
ause of the overlap with signs of preeclampsia and the differing prognoses of th
e two conditions. Preeclampsia is a complication that presents after 20 weeks of
gestation, which suggests the abnormal proteinuria in this patient is associate
d with another disease. The finding of 2+ protein on dipstick urinalysis is not
specific, but the hematuria and erythrocyte casts suggest underlying glomerulone
phritis rather than preeclampsia.
4- A 50-year-old man who has had diabetes mellitus for 12 years is concerned abo
ut becoming dependent on dialysis. His disease has been inadequately controlled
(hemoglobin A1C range 8.7% to 11.8%). His urinalysis is strongly positive for pr
otein, and his serum creatinine concentration has increased from 1.2 to 1.9 mg/d
L in the past 8 months. His blood pressure has been 150/90 mm Hg and on occasion
as high as 210/120 mm Hg. Which treatment is most important to reduce the proce
ss of this patient s kidney disease? A. Insulin B. Antihypertensive agent C. HMG
-CoA reductase inhibitor ("statin") D. Protein-restricted diet E. Aspirin Answer
is B. This patient has already started on the path to progressive diabetic neph
ropathy. Although all of the preventive measures still warrant attention, blood

pressure control at this stage is most important. From the onset, good glycemic
control (hemoglobin A1C 7% to 7.5%), a protein-regulated diet to reduce intraglo
merular pressure as well as antihypertensive therapy with an ACE-inhibitor shoul
d be considered.Antihypertensive therapy attenuates the decline in renal functio
n in patients with all forms of diabetes.
5- A 26-year-old patient is presented with amenorrhea of 2.5-month duration. Lab
analysis indicates increased levels of thyroid and cortisol binding proteins. T
here is an elevated total cortisol and ACTH. Free cortisol level is within norma
l range. These findings are most likely suggestive of which of the following con
ditions? A. Hypothyroidism B. Addison s disease C. Conn s disease D. Cushing s d
isease E. Pregnancy answer is E.
Two weeks following a viral illness, a teenage boy breaks out in an evolving ras
h that is remarkable for target lesions. What is the primary treatment? a. Epine
phrine b. Glucagon c. Corticosteroids d. Antihistamines e. Symptomatic or suppor
tive therapy depending on severity. Answer is e. its erythema multiform
The most prevalent of allergic disease in school-age children is: a. Atopic derm
atitis b. Food allergy c. Asthma d. Allergic rhinitis e. Drug allergy Answer is
d.

Cheilosis and glossitis are features of ( more than one answer ): a. vit A def.
b. riboflavin def. c. vit. C def. d. pyridoxine def. e. vit. E def. Answers are
b&d. Which one is the most common adverse effect of intranasal steroids? a. Nasa
l irritation b. Septal perforation c. Nasal bleeding d. Short stature e. Adrenal
suppression Answer is a.
Increased risk for intussusception was observed as a rare complication following
immunization with which vaccine? a. IPV b. OPV c. Rotavirus vaccine d. HAV e. H
BV Answer is c.
A 60 y/o Japanese man visiting US with excellent health until 6 months ago, when
he first noted mild upper abdominal fullness after meals. On P/E hyperpigmented
, heaped-up velvety lesions in the neck, axillae, and groin is noted. Which of t
he following conditions is associated with the skin findings? a- Non-Hodgkin s l
ymphoma b- Anorexia nervosa c- Acute leukemia d- Adenocarcinoma of the stomach

e- Addison s disease
Answer is d, skin lesion is acanthosis nigricans.
Which of the following should be done annually after age 40 in the asymptomatic,
average- risk man in order to promote the early detection of cancer? a- Colonos
copy b- Sigmoidoscopy c- Digital rectal examination with palpation of the prosta
te d- Digital rectal examination with palpation of the prostate and stool guaiac
e- Digital rectal examination with palpation of the prostate, stool blood test,
and chest x-ray
Answer is c.
During a routine checkup, a 65 y/o man is found to have a level of serum Alk Ph.
three times the upper limit of normal. Serum Ca and ph. concentrations and LFT
results are normal. He is asymptomatic. The most likely diagnosis is a- metastat
ic bone disease b- primary hyperparathyroidism c- occult plasmacytoma d- Paget s
disease of bone e- osteomalacia answer is d.
Which of the following is NOT a predisposing factor for the development of a her
nia? a- Ascites b- Obesity

c- Cystic fibrosis d- Chronic obstructive pulmonary disease e- Peritoneal dialys


is answer is b. Ascites, peritoneal dialysis, ventriculoperitoneal shunt, cystic
fibrosis, and chronic obstructive pulmonary disease all predispose patients to
hernia formation because they increase intraabdominal pressure. Other risk facto
rs include a positive family history, undescended testis, and genitourinary abno
rmalities.
A 5-month-old infant has had several episodes of wheezing, not clearly related t
o colds. The pregnancy and delivery were normal; the infant received phototherap
y for 1 day for hyperbilirubinemia. He had an episode of otitis media 1 month ag
o. There is no chronic runny nose or strong family history of asthma. He spits u
p small amounts of formula several times a day, but otherwise appears well. His
growth curve is normal. An examination is unremarkable except for mild wheezing.
Which one of the following is the most likely diagnosis? A) Benign reactive air
way disease of infancy B) Cystic fibrosis C) Unresolved respiratory syncytial vi
rus infection D) Early asthma E) Gastroesophageal reflux Answer is E, gastroesop
hageal reflux is a common cause of wheezing in infants. At 5 months of age, most
infants no longer spit up several times a day, and this is a major clue that th
e wheezing may be from the reflux. Also, there is no family history of asthma an
d the wheezing is not related to infections. Cystic fibrosis is more likely to p
resent with recurrent infections and failure to thrive than with intermittent wh
eezing.
*** ECG findings of pulmonary edema include all of the following except: a- deep
S1 b- depressed ST in lead I & II c- prominent Q1 & inversion of T3 d- left axi
s deviation e- clockwise rotation in the precordial leads answer is d. it cause
Rt. axis deviation.
*** chest pain & friction rub 3 days after admission to ICU, indicate which of t
he ffg? a- misdiagnosis of infarction

bcdechest trauma viral infection transmural infarction dissecting aneurysm


answer is d.
*** Neuropathy secondary to gout manifests as: a- NS b- ARF c- ATN d- Isosthenur
ia & moderate albuminuria e- Malignant HTN Answer is d.
*** Tx of choice for cutaneous manifestation of protoporphyria is: a- phenobarb.
b- Corticosteroid c- High carb. Diet d- Beta caroten e- Chlorpromazine Answer i
s d.
*** 25 y/o female with intermittent double vision, on CXR an Ant. Mediastinal ma
ss is noted. Whats the next step of Mx? a- serum Ca messurement b- brain MRI c- e
valuation of T cell function d- serum gamma globulins messurement e- order GTT a
nswer is d. association of thymoma and myasthenia gravis. 5-10% of Px with thymo
ma have low serum gamma globulins.
*** 76 y/o male with 80 pack/y smoking Hx, was diagnosed with lung cancer 4 m ag
o, hes brought to hospital in state of coma, his serum ca= 16 mg/dl, which of the
ffg, would be the most useful to reduce ca rapidly: a- acetazolamide b- furesmi
de c- hydrochlorothiazide

d- manitol e- spironolactone answer is b- furesmide which increase urinary excre


tion of ca.
*** 50 y/o man with Hx of smoking & cough for the past 2 months, is found to hav
e a solitary pulmonary nodule with no mediastinal dis on CXR. Hes expected to hav
e best prognosis if he has which type of ffg tumors? a- small cell carcinoma bpoorly diff. Adenocarcinoma c- metastatic carcinoma d- well diff. SCC e- well di
ff. Adenocarcinoma answer is e- well diff adenocarcinoma
*** 27 y/o woman presents with sudden pain & decrease in hearing in Rt. ear whil
e picking it with matchstick. On P/E, a traumatic rypture of Rt. tympanic membra
ne & some blood clot is seen. Whats the best advice or procedure? a- advice her t
o use earplug while shampooing & showering b- advice her to clean the ear canal
with Q-tips c- remove the blood clot d- antibiotic eardrops e- oral + eardrop an
tibiotics answer is a- advice her to use earplug while shampooing & showering
*** 3 days after hospital admission wih Dx od acute pancreatitis, a 45 y/o man h
as 5 of the ranson criteria, On P/E, T=102, BP=120/70, PR= 130, RR= 18, Px is in
obvious distress, abdomen is distended with no audible bowel sounds, tenderness
on upper abd. without any rebound tenderness is noted. Whats the best next step
to confirm the Dx? a- serum amylase b- paracentesis c- ERCP d- Dynamic CT scan e
- Laparascopy f- US g- Abdominal x-ray Answer is d- dynamic CT scan is the best
investigative way for pancreatic necrosis.

*** A 65 y.o. man with ischemic heart disease comes to the ER in acute CHF. The
ophthalmic medicine most likely to be responsible is a. Acetazolamide b. Pilocar
pine c. Tobridex d. Trifluoridine e. Timolol maleate Answer is e- timolol maleat
e.
*** A 40 y.o. w/spastic bladder is treated w/ anticholinergics. The pt. will com
plain of: a- Diplopia b- Decreased distance vision c- Decreased near vision d- D
ecrease in color vision e- Snowy, sparkly vision Answer is c- decreased near vis
ion
*** Which of the following lab values would be most consistent with a woman who
has a Hx of lung cancer and has been on chemotherapy for several weeks.The patie
nt now presents with acute anemia. a. reticulocyte count of 60 x 109 b. plasma H
gb of 7.2 g/dl c. Hct of 34% d. MCV of 70 fL e. RBC of 5.9 million/mm3 answer is
a.
*** A healthy 12-month-old boy has a brother with adenosine deaminase deficiency
. Which of the following vaccines should you NOT administer to the infant? (A) D
iphtheria-tetanus-pertussis (DTP) (B) H. influenzae (HiB) (C) Hepatitis B (Hep B
) (D) Oral polio vaccine (E) Measles-mumps-rubella (MMR)

answer is D- The main problem in this scenario is administering a live, attenuat


ed vaccine that may be followed by transmission to the immunocompromised sibling
and cause severe, possibly fatal, disease. Of the attenuated vaccines, the oral
polio vaccine is the most dangerous because the virus is shed in the feces, whi
le attenuated vaccines administered by inoculation (such as measles-mumpsrubella
), which are not shed by the recipient, are considerably less likely to be trans
mitted from person to person. The other three vaccines are not live vaccines and
have no special risk in an immunocompromised individual
*** A 30-year-old man sustains brain damage as the result of an automobile accid
ent. Neurologic examination reveals incomplete retrograde amnesia and severe ant
erograde amnesia as well as inappropriate social behavior, including hyperphagia
, hypersexuality and general disinhibition. The brain injury would most likely i
nvolve the: (A) frontal lobes, lateral convexity. (B) frontal lobes, medial surf
ace. (C) temporal lobes, lateral convexity. (D) temporal lobes, medial surface.
(E) thalami. Answer is D- Bilateral damage of the medial temporal gyri, includin
g the amygdalae, may cause severe memory loss (hippocampal formations). Such dam
age to the amygdalae may lead to inappropriate social behavior (e.g., hyperphagi
a, hypersexuality, general disinhibition). Bilateral destruction of the amygdala
e results in the Klver-Busy syndrome.
*** A 24 y/o college student, G1P1 presents for her annual pelvic examination an
d Pap smear, and renewal of her ocp. Hx & P/E are unremarkable. Her Pap smear is
subsequently reported as high-grade squamous intraepithelial lesions (HGSIL). T
he most appropriate next step in this patient s management is (A) Repeat Pap sme
ar in one year. (B) Repeat Pap smear now. (C) Colposcopy with ECC and directed b
iopsy. (D) Excisional biopsy. (E) Cervical conization. Answer is C. HGSIL on scr
eening examination requires histologic diagnosis so that appropriate treatment c
an be selected. Repeat Pap smear is another screening test and is thus inappropr
iate. Conization for diagnostic purposes is needed only if colposcopy and biopsy
prove inadequate.

*** A 63 y/o man with no significant past medical history is admitted with an ac
ute abdomen secondary to gastric perforation. If a definitive ulcer procedure is
performed on this patient, which of the following procedures is associated with
the lowest ulcer recurrence rate? (A) Truncal vagotomy (B) Truncal vagotomy and
pyloroplasty (C) Vagotomy and antrectomy (D) Parietal cell vagotomy (E) Gastric
bypass answer is c. Among the choices provided, vagotomy with a simultaneous an
trectomy has the lowest ulcer recurrence rate. However, this procedure is associ
ated with a higher morbidity secondary to anastomotic complications (e.g., leak
or stricture) or problems associated with denervation of the proximal stomach an
d distal bowel. Complications include postvagotomy diarrhea, dumping syndrome, d
elayed gastric emptying and alkaline reflux gastritis.
*** A 16 y/o boy presents to his physician with a history suggestive of systemic
lupus erythematosus. The patient has a rash, arthralgia and mild proteinuria. S
erologic evaluation detects no antibodies of diagnostic value. Which one of the
following diseases is most likely to be diagnosed in this patient? (A) Rheumatoi
d arthritis (B) Common variable hypogam-maglobulinemia (C) C2 deficiency (D) Sjgr
en s syndrome (E) Wegener s granulomatosis answer is c. A deficiency in compleme
nt component C2 may manifest itself as a disorder similar to systemic lupus eryt
hematosus, possibly because of a failure of complementdependent mechanisms to el
iminate immune complexes. Most patients with rheumatoid arthritis have several a
ntibodies (most notably the anti-IgG antibody known as rheumatoid factor). Simil
arly, Sjgren s patients are diagnosed by detection of antibodies to exocrine glan
d duct epithelium. Patients with Wegener s granulomatosis have antineutrophil cy
toplasmic autoantibodies. Common variable hypogammaglobulinemia will evidence it
self by the occurrence of repeated bacterial infections.

1- a 4 y/o child presents with upper respiratory illness. P/E reveals mental ret
ardation, eczema, hypopigmentation and blue eyes. Whats the most likely Dx? a- Do
wn syn. b- Tuberous sclerosis c- PKU d- Osteogenesis imperfecta e- Cretinism fGalactosemia Answer is PKU.
2- 28 y/o male IV drug abuser presents with painful, erythematous nodules on fin
gerpad and linear hemoorhage beneath the fingernail of indexfinger. On P/E a gra
de II, high pitched diastolic murmur is heard on 2nd & 3rd Rt. ICS. Whats these l
esion on the hand are related to? a- immunocmplex vasculitis b- thrombocytopenia
c- coagulation factor def. d- DIC Answer is a. Px has infective endocarditis du
e to staph, aureus involving aortic valve.the nodules are oslers nodes and nailbe
d finding is splinter hemorrhage and both are example of immunocomplex vasculiti
s.
3- 53 y/o woman presents with SOB and weight loss despite eating well, she also
have intermittent episodes of heart racing. On P/E, multinodular thyroid and war
m and moist palm are noted. Hr=106/min, whats the most likely Dx? a- graves dis. b
- Papillary adenocarcinoma c- Plummers dis. d- Follicular adenoma Answer is c ( o
r toxic nodular goiter ), its hyperthyroidism associated with multinodular goiter
.
4- on a routine examination of a 5 y/o female a heart murmur is heard. Its medium
pitched, systolic ejection murmur, with musical quality. Which of the ffg is th
e characteristic of this murmur: a- its best heard along the lower Lt. & midstern
al border b- its common in infancy c- its best heard while Px is lying down on the
Lt. d- its less intense with fever or excitement

answer is A. an innocent murmur, which is best heard in supine position. It occu


rs only in systole ( never in diastole ), uaually grade I or II, and heard on LS
B in children between ages 3-7 ( rarely in infancy ).
5- Which of the ffg is consistent with an adult Px with RA & salysilate intoxica
tion: PH PCO2 Bicarb a- 7.29 53 25 b- 7.38 22 12 c- 7.53 49 39 d- 7.43 70 46 e7.28 28 12 answer is b. respiratory alkalosis+metabolic acidosis
** 32 y/o athelete presented with jaundice.no previous liver disease.investigati
on showed elevated Alk Ph.,no viral infection and no stones. what is ur presumpt
ive diagnsis and possible cause? cholestatic jaundice due to methyl testosterone
administration.
** 35 y/o woman had an attack of billiary colic. which analgesic is contraindica
ted and why? morphine increase intrabilliary pressure which worsens the pain.
1- A 70-year-old African-American male who has been hospitalized for 2 weeks for
congestive heart failure develops severe, persistent diarrhea. For the past 3 da
ys he has had abdominal cramps and profuse semi-formed stools without mucus or b
lood. The patients current medications include captopril (Capoten), digoxin, furo
semide (Lasix), subcutaneous heparin, spironolactone (Aldactone), and loperamide
(Imodium). He has coronary artery disease, but has been relatively pain free si
nce undergoing coronary artery bypass surgery 4 years ago. An appendectomy and c
holecystectomy were performed in the past, and the patient has since been free o
f gastrointestinal disease. On physical examination his blood pressure is 100/80
mm Hg, pulse 100 beats/min and regular, and temperature 37.0 C (98.6 F). He has m
ild jugular venous distention and crackles at both lung bases. Examination of hi
s heart is unremarkable, although there is 1+ dependent edema. His abdomen is di
ffusely tender without masses or organomegaly. Rectal examination is normal. The
results of routine laboratory tests, including a CBC, chemistry profile, EKG, a
nd urinalysis, are all normal. The stool examination shows numerous white blood
cells. Of the following, the most likely diagnosis is

A) viral gastroenteritis B) Clostridium difficile colitis C) ulcerative colitis


D) gluten-sensitive enteropathy (celiac sprue) E) digoxin toxicity This patient
most likely has Clostridium difficile colitis, suggested by semiformed rather th
an watery stool, fecal leukocytes (not seen in viral gastroenteritis or sprue),
and a hospital stay greater than 2 weeks. While this disease has traditionally b
een associated with antibiotic use, it is posing an increasing threat to patient
s in hospitals and chroniccare facilities who have not been given antibiotics. T
he primary sources for infection in such cases have been toilets, bedpans, floor
s, and the hands of hospital personnel. Prompt recognition and treatment is esse
ntial to prevent patient relapse and to minimize intramural epidemics. The diarr
hea of ulcerative colitis usually contains blood and occurs intermittently over
a protracted course. Digoxin toxicity is likely to be accompanied by electrocard
iographic and laboratory abnormalities, particularly hyper- or hypokalemia.
2- A 79-year-old man is admitted to the medical ward 3 days status post subdural
hematoma drainage, C3 cervical spine fracture, and fixation of multiple extremi
ty fractures sustained in a motor vehicle accident. The patient is now awake and
oriented to person, place, and time, but is a lower cervical spine incomplete q
uadriplegic. Physical examination reveals some minimal sensation in the legs, bu
t no ability to move the extremities. There is a Foley catheter in place that is
draining yellow colored urine. Doppler ultrasonography demonstrates a thrombus
in the left popliteal vein. The most important next step in the management of th
is patient is A. daily Doppler ultrasonography of the lower extremities B. infer
ior vena cava filter placement C. subcutaneous heparin D. tissue plasminogen act
ivator thrombolysis E. warfarin F. weekly ventilation/perfusion scans for a pulm
onary embolus The correct answer is B. This patient has documented deep venous t
hrombosis (DVT) on ultrasonography and has had recent intracranial surgery. Intr
acranial surgery is an absolute contraindication to anticoagulation. Because the
patient has a documented DVT, an inferior vena cava filter is necessary to prev
ent potentially fatal pulmonary emboli.

Daily ultrasonography of the lower extremities (choice A) is a way to follow the


extent of the documented thrombus in the left popliteal vein. It is not suffici
ent to simply follow the extent of the clot, however. The known DVT puts him at
a risk for a pulmonary embolus, and therefore he needs an inferior vena cava fil
ter. Anticoagulation with subcutaneous heparin (choice C) is absolutely contrain
dicated as the patient has had recent neurosurgery. Thrombolysis (choice D) will
treat the current thrombus in the left popliteal vein, but it will not prevent
further thromboses from occurring. Thrombolysis with tissue plasminogen activato
r is also contraindicated so soon after intracranial surgery. Anticoagulation wi
th warfarin (choice E) is absolutely contraindicated as the patient has had rece
nt neurosurgery. A screening study for pulmonary emboli (choice F) is inadequate
for this high-risk patient. Although both ventilation/perfusion scanning and CT
pulmonary angiography are effective studies to diagnosis a pulmonary embolus, t
his patient needs definitive treatment to prevent a pulmonary embolus.
3- A 23-year-old college student comes to the clinic because of odynophagia with
solids and liquids and dysphagia that is most severe when eating solid foods. T
he patient had a past medical history of Shigella colitis last year while she wa
s a Peace Corps volunteer in Peru. She takes oral contraceptives and smokes 1 pa
ck of cigarettes daily. She does not drink alcohol. Vital signs are: temperature
37.8 C (100 F), blood pressure 100/70 mm Hg, pulse 79/min, and respirations 8/m
in. Physical examination is normal. Electrocardiogram reveals normal sinus rhyth
ms with a rate of 85/min and a markedly enlarged QRS complex in leads V3-V5. Che
st x-ray reveals an enlarged cardiac silhouette. A barium esophagram demonstrate
s a tapering of the distal esophagus that eventually releases as the esophagus i
s distended. There is no evidence for extrinsic or intrinsic compression of the
distal esophagus or an esophageal mass. There is no reflux. The test most likely
to lead to a unifying diagnosis in this case is A. an agglutination test for tr
ypanosomes B. a chest CT C. an esophageal manometry D. a liver biopsy E. a myoca
rdial biopsy

The correct answer is A. The findings of achalasia and cardiomyopathy in a patie


nt with history of travel to Central or South America support the diagnosis of C
hagas disease. This is caused by infection of Trypanosoma cruzi in the muscles o
f the heart, esophagus, and colon. A serum agglutination test is a noninvasive m
eans of testing for this infection, and is highly sensitive. Left untreated, hea
rt failure and megacolon could develop. A chest CT (choice B) would not reveal a
ny specific findings of Chagas disease. Using a CT to evaluate for an occult can
cer causing esophageal narrowing is superfluous given the findings on the barium
esophagram, which is specific for achalasia. An esophageal manometry (choice C)
is a confirmatory test for achalasia. Given the highly suggestive findings of a
chalasia on the barium esophagram, manometry would not be necessary. A liver bio
psy (choice D) has no role in the evaluation of Chagas disease. A myocardial bio
psy (choice E) is not necessary to diagnose Chagas disease with the availability
of the serum agglutination test for trypanosomes. Invasive procedures should be
deferred until they are absolutely necessary.
4- A 17-year-old girl is brought to the office by her mother because she has mis
sed many periods. The girl admits to binge eating and exercising in order to preve
nt weight gain. She tells you that she is definitely not pregnant, because she h
as not had any sexual relations in the past 11 months and thinks she is not gett
ing her menstrual period because of the excessive physical exercise she has been
doing in the past several weeks. Physical examination is significant for bradyc
ardia and significant weight loss compared to the last year. A pregnancy test is
negative.At this time you should order A. amylase B. BUN and creatinine C. live
r function tests D. serum potassium E. thyroid function tests The correct answer
is D. Bulimic patients frequently engage in compensatory behaviors to prevent w
eight gain. Those include self-induced vomiting, abuse of diuretics, laxatives,
enemas, or diet pills. The metabolic disorders frequently seen in these patients
are, mostly hypokalemia and hypomagnesemia.

Amylase (choice A) is not a routine test ordered in bulimic patients. If there h


as been evidence of long starvation and suspicion of other medical conditions, i
ts level might alter. BUN and creatinine (choice B) can be increased if there ar
e signs of dehydration, secondary to the abuse of diuretics. Otherwise, these va
lues should not be changed significantly. Liver function tests (choice C) are us
ually not changed in bulimic patients. They can be ordered to rule out other med
ical conditions if necessary. Thyroid function tests (choice E) should be done a
s a part of regular workup of patients presenting with this clinical picture. It
is not, however, the first to be ordered.
5- A 76-year-old woman is brought to the hospital by her son because of "rapid b
reathing." She has advanced Alzheimer disease and is unable to give a coherent h
istory. She was recently diagnosed with breast cancer. She lives alone, but norm
ally has a health care aide during the day. The aide was not available when the
son tried to reach her to ask if anything happened. The son has not seen his mot
her in 2 months. An accentuated fall in systolic blood pressure during inspirati
on would most likely suggest A. anxiety B. cardiac tamponade C. myocardial infar
ction D. senile aortic stenosis E. sepsis The correct answer is B. This question
describes pulsus paradoxus, which is when there is an accentuated fall in systo
lic blood pressure (>10 mm Hg) during inspiration. There is usually a small decr
ease in blood pressure during inspiration, but it is exaggerated in cardiac tamp
onade because the external compression caused by fluid accumulation around the h
eart leads to impaired ventricular filling, reduced left ventricular stroke volu
me, and a reduction in systolic blood pressure. Cardiac tamponade can occur acut
ely after trauma or develop chronically from the accumulation of pericardial flu
id malignancies (she has breast cancer), uremia, infections, collagen vascular d
iseases, or radiation.

All of the other choices, anxiety (choice A), myocardial infarction (choice C),
senile aortic stenosis (choice D), and sepsis (choice E), may be associated with
tachypnea, but are not usually direct causes of pulsus paradoxus.
*** A 5 y/o boy is brought to the clinic because of a fever for 5 days and a sor
e throat and malaise. The mother tells you that he is usually a very healthy chi
ld and he is up to date on all of his immunizations. Besides the mother, he live
s at home with an older brother and sister, neither of them are sick. On P/E, T=
39.5 C (103.1 F), he has a peeling rash on his extremities, one 2 cm lymph node
on the right anterior cervical chain, a confluent truncal rash, and mild conjunc
tivitis. Appropriate management is taken. The most important long-term Mx of thi
s child is: A. antibiotic prophylaxis to prevent rheumatic fever B. echocardiogr
ams to look for coronary artery aneurysms C. excisional biopsy and surveillance
of lymph nodes for malignancy D. nothing, as this is a case of scarlet fever and
he will completely recover E. serial lumbar punctures The correct answer is B.
The vignette demonstrates a case of Kawasaki disease which is characterized by h
igh fever for >5 days, unilateral cervical lymph node, macular papular rash to t
ruck, peeling of hands and feet, and conjunctivitis. The most important sequela
of Kawasaki is the development of coronary aneurysms, and so children need to be
monitored with EKGs and echocardiograms for at least 2-3 months after the acute
illness. Aneurysms form from 7-45 days after illness onset. Streptococcal phary
ngitis is characterized by erythema of the oropharynx, palatal petechiae, and ce
rvical lymphadenopathy. There is no peeling, rash or conjunctivitis. Antibiotics
are used for treatment and rheumatic fever prophylaxis (choice A). The enlarged
lymph node is a reactive lymphadenopathy and need not be biopsied or excised (c
hoice C). Upon proper treatment for KD, lymphadenopathy regresses. This is not a
case of scarlet fever (choice D) which is characterized by exanthem and a fine,
sandpapery rash, and is usually caused by Streptococcus. Although children with
KD often present with a toxic appearance, lumbar punctures (choice E) is not th
e standard of care and need not be performed.

*** A 57-year-old woman with a long history of Crohn s disease comes to the offi
ce because of slowly progressive ataxia and paresthesias. She is currently off a
ll medications. Physical examination shows a decreased vibratory and positional
sense and mild ataxia. A CT of the head is unremarkable. Laboratory studies show
:
You suspect B12 deficiency secondary to Crohn s disease. A Schilling test is per
formed. You expect the study to show: In order: Radiolabeled Vitamin B12 urinary
excretion, Radiolabeled B12 excretion after intrinsic factor added , Radiolabel
ed B12 excretion after intrinsic factor + antibiotics A. Normal / Normal / Norma
l B. Low/ Normal/ Normal C. Low /Low/ Low D. Low /Low/ Normal E. Normal /Low/ No
rmal The correct answer is C. This patient likely has a lesion in her ileum from
Crohn s disease. This is the area where B12 is preferentially absorbed. If ther
e is a lesion in the ileum, the Schilling test will reveal no ability to absorb
B12. An understanding of the Schilling test should facilitate answering this que
stion. If B12 deficiency develops, a Schilling test can help to define why this
happened. The first step is to saturate the body with B12 by giving a large IV d
ose of normal B12. Simply, this ensures that any additional B12 absorbed will be
excreted in the urine. This patient does not lack intrinsic factor so adding IF
will not improve B12 absorption. Additionally, this patient s problem is not th
e lack of dietary B12, so giving B12 alone could not correct the underlying prob
lem.
*** You are asked to see a 3 wk/o infant in the ED with a 1-day history of fever
. The parents measured his temperature because he "felt warm" to them and found
a temperature of 38.3 C (101.0 F). He has been feeding normally, taking 2 ounces
of formula every 3-4 hours. He had 6 wet diapers the previous day. Examination
shows an

active infant with a temperature of 38.8 C (101.8 F). His skin perfusion is good
and his physical examination, including examination of his tympanic membranes,
is normal. There are no ill household contacts. The most appropriate next step i
s to A. discharge the patient with close outpatient follow up B. inquire about t
he mother s group B streptococcal status at delivery C. obtain the infant s vacc
ination history D. order a urinalysis and, if negative, do blood and CSF culture
s E. send blood, urine, and CSF cultures and begin empiric intravenous antibioti
c therapy The correct answer is E. There is absolutely no way to reliably distin
guish a self-limited viral illness from sepsis or meningitis in an infant less t
han 4-weeks of age. Accordingly, all infants with a fever greater than 38 C (100
.4 F) in this age group require full evaluation, including admission and parente
ral antibiotics. The incidence of sepsis is somewhere between 5-10% with this de
gree of fever and can be catastrophic if missed, resulting in death or permanent
neurologic disability. Discharging the patient (choice A) is inappropriate beca
use well-appearing infants in this age group may still have a potentially lethal
bacterial disease. At 3 weeks, this infant is at the peak incidence of late ons
et group B streptococcal disease, but the mother s group B streptococcal status
(choice B) does not correlate well with occurrence of late onset group B strepto
coccal disease and so it is not relevant here. Maternal group B streptococcal sc
reening is useful in the management of early-onset disease (disease within the f
irst few days of life). At 3 weeks this infant is unlikely to have received any
vaccinations (choice C), with the possible exception of hepatitis B vaccine. He
would not in any case have received Haemophilus influenzae or conjugated Strepto
coccus pneumoniae vaccine, which are the only two that could potentially influen
ce his susceptibility to sepsis or meningitis. Urinalysis (choice D) is not reli
able in excluding urinary tract infection in very young infants, and therefore i
s not a good screening test to decide if further evaluation is necessary.
*** A 30 y/o woman is brought to the emergency department after a high-speed MVA
in which she was an unrestrained driver. She was conscious at the scene with a
Glasgow Score of 13. There were beer bottles evident in the car. On arrival at t
he emergency department, primary survey shows an abdominal bruise on the RUQ. He
r vital signs are

stable and blood alcohol level is 2449 mg/dL. Her PMH is remarkable for alcoholi
c pancreatitis twice in the previous 3 years. The patient complains of marked ri
ght upper quadrant pain and has significant tenderness on palpation of her right
upper and right lower quadrants. An abdominal CT scan shows pericholecystic flu
id and possible gallbladder wall thickening. The most appropriate next step to d
iagnose possible gallbladder rupture is: A. diagnosis can be made by CT scan alo
ne B. ERCP C. HIDA scan D. exploratory laparoscopy E. exploratory laparotomy The
correct answer is D. This patient has suffered a traumatic injury to her right
upper quadrant and there is a clinical suspicion of gallbladder rupture. The gol
d standard for such a diagnosis is direct visualization. Since laparoscopy is wi
dely available, it has become the modality of choice to undertake direct visuali
zation of the suspected injury. The diagnosis cannot be made by CT alone (choice
A). The CT shows only pericholecystic fluid. This is consistent with chronic or
acute cholecystitis, both of which are a possibility given her history of alcoh
ol use and pancreatitis. The issue, given her trauma, is whether the gallbladder
has ruptured. An ERCP (choice B) is an endoscopic diagnostic and therapeutic to
ol for the management of biliary disease. It has no role in the diagnosis of gal
lbladder rupture. It can however be used to demonstrate gallbladder filling with
contrast after the removal of biliary stones. A HIDA scan (choice C) is an imag
ing modality utilizing radioactive tracers to visualize the gallbladder. However
, given the widespread use of laparoscopic equipment, this test has been largely
supplanted by the new, more sensitive diagnostic laparoscopy.
Exploratory laparotomy (choice E) has attendant intra- and postoperative complic
ations that make it largely reserved for centers where laparoscopy is not availa
ble. It is also reserved for patients with extensive abdominal trauma or clinica
l signs and symptoms consistent with massive abdominal pathology where laparosco
py would not be useful or for patients that require a laparotomy for another ind
ication.

*** A 64 y/o man comes to the clinic because of a "spot" on the side of his face
that has been there for about 8 months. He says that he is an executive at a lo
cal company and is retiring at the end of the year. The company has hired a port
rait artist to paint his picture that will hang in the boardroom for many years
to come, and so he realized that this is a good time to "have this thing taken o
ff." He thinks that the lesion has not grown since he noticed it, but he has not
paid it much attention. He plays tennis every weekend at his country club and t
hen lies in the sun with his wife. This is the first time you have seen this pat
ient, but he tells you that he has been very healthy and has only suffered throu
gh "a couple of bouts of kidney stones" over the years. Physical examination sho
ws a 2.3-cm waxy, verrucous, dark brown papule with a stuck-on appearance.The most
likely diagnosis is A. actinic keratosis B. basal cell carcinoma C. dermatofibr
oma D. melanoma E. psoriasis F. seborrheic keratosis G. squamous cell carcinoma
The correct answer is F. This patient most likely has seborrheic keratosis. This
lesion is characterized by light brown to black papules or plaques with an adhe
rent waxy, greasy scale. The "stuck-on" appearance is very characteristic. It is
most often found on the face and trunk.
A 35 y/o white female is brought to your office after a brief loss of consciousn
ess. No tonic-clonic activity was observed, nor did the patient experience urina
ry incontinence. She recalls that about 10 seconds prior to blacking out she fel
t nauseated, began sweating, and became pale. When she recovered, she had no mem
ory loss or confusion. Her physical examination, including a thorough neurologic
examination, is unremarkable. A CBC, blood profile, and EKG are all within norm
al limits. Appropriate management at this time would include a.an EEG b.24-hour
Holter monitoring c.echocardiography d.a CT scan of the head e.reassurance

e- Patients with vasovagal syncope often present with prodromal symptoms such as
nausea, diaphoresis, pallor, and lightheadedness. This often occurs in stressfu
l or frightening situations. These premonitory symptoms often occur before the l
oss of consciousness. A quick recovery of mental function after an episode of lo
ss of consciousness makes it much more likely that this was a syncopal episode r
ather than a seizure, as seizures are often followed by a postictal period of re
sidual confusion. A patient does not always have to experience tonic-clonic move
ments for the diagnosis to be a seizure disorder, since brief tonic-clonic movem
ents can be seen in some syncopal episodes. However, patients who experience an
aura or incontinence of urine or feces often have had a seizure.
For a pregnant woman in the 26th week of gestation who tested positive for Strep
tococcus B you should start the treatment: a.right away b.one week before delive
ry c.24 hours before delivery d.during delivery d- Physicians who culture for GB
S carriage during prenatal visits should do so late in pregnancy (35-37 weeks ges
tation); cultures collected earlier do not accurately predict whether a mother w
ill have GBS at delivery.
You diagnose acute pancreatitis in a 45-year-old white male. As you initiate the
rapy and closely monitor his progress, which one of the following complications
is most likely to develop? a.Hypomagnesemia b.Hypoglycemia c.Hypercalcemia d.Acu
te hypertension a- Treatment strategies for pancreatitis vary somewhat and are r
elated to the degree of inflammation. In patients with mild pancreatitis, a ther
apeutic regimen of avoidance of oral intake, intravenous hydration, and analgesi
a usually suffices. However, patients with more severe pancreatitis are likely t
o develop significant complications, and require closer monitoring. Patients wit
h hypotension and vascular instability frequently require massive fluid resuscit
ation. Patients with metabolic complications such as hyperglycemia, hypocalcemia
, or hypomagnesemia may require insulin, calcium, or magnesium supplementation.

Immediate gastric lavage is contraindicated in treating acute ingestion of which


one of the following? A.Salicylates B.Strychnine C.Ethanol D.Acetaminophen E.Ph
enothiazines b- Ten to 30 minutes after ingestion of strychnine, untoward sympto
ms begin. Often without any warning the patient falls into violent convulsions.
Gastric lavage is postponed until treatment designed to prevent the convulsions
is started. Initiation of gastric lavage as soon as possible is indicated in tre
ating poisoning by salicylates, ethanol, acetaminophen, and phenothiazines.
Which one of the following depressed patients is most likely to commit suicide?
a.A 26-year-old male who repeatedly denies any thoughts of suicide b.A 30-year-o
ld female who has been hospitalized overnight on several occasions for attempted
suicide c.A 50-year-old recently divorced alcoholic male who feels life is hope
less d.A 50-year-old female who thinks of suicide and fears she might act on her
thoughts c- Assessment of suicidal risk is critical in determining the need for
and duration of hospitalization of depressed patients. Most suicides are planne
d, not impulsive, and carried out successfully most often by the elderly, males,
those in poor health, alcoholics, schizophrenics, those who have recently lost
a loved one (especially a mate), and those suffering from depressive disorders.
Many depressed patients think about suicide, and a physician should take these p
atients seriously; however, among this group of patients, the 50-year-old recent
ly divorced alcoholic male has the highest risk of successful suicide.
Which disease could be with higher BP in lower extremity than in upper extremity
? TAKAYASO: High in the lower
A 70-year-old former shipyard worker, who smokes one pack of cigarettes daily, n
otes progressive weight loss and debility over a period of 6 months. Over a peri
od of 1 month, he develops right chest pain and a nonproductive cough. A chest x
-ray reveals extensive pleural thickening, pleural effusion, and hilar adenopath
y. A pleural biopsy confirms mesothelioma. Which of the following is a favorable
prognostic factor in this patient? a. Male sex b. Extent of disease at diagnosi
s c. Good performance status d. Pain as presenting symptom

e. Age over 65 f. None of the above f- All of the clinical components noted exhi
bit very poor prognostic factors in the patient with mesothelioma, a particularl
y aggressive disease that remains unresponsive to many forms of aggressive thera
py.
Which one of the following has proven most useful for breast cancer screening in
women who have had silicone breast implants? a.Thermography b.Ultrasonography c
.Mammography d.Magnetic resonance imaging c- Screening by conventional film-scre
en mammography, supplemented by the displaced or Eklund view, is the recommended
method for breast cancer screening in women with breast implants. The displaced
view draws the breast forward while placing the implant posteriorly, increasing
the amount of breast tissue visualized. None of the other modalities has proven
useful for screening asymptomatic women for breast cancer.
A 55-year-old woman has had profuse watery diarrhea for 3 months. Laboratory stu
dies of fecal water show the following: Sodium: 39 mmol/L Potassium: 96 mmol/L C
hloride: 15 mmol/L Bicarbonate: 40 mmol/L Osmolality: 270 mosmol/kg H2O (serum o
smolality: 280 mosmol/kg H2O) The most likely diagnosis is A villous adenoma B l
actose intolerance C laxative abuse D pancreatic insufficiency E nontropical spr
ue The answer is A In the case described, the osmolality of fecal water is appro
ximately equal to serum osmolality. Furthermore, there is no osmotic "gap" in th
e fecal water; the osmolality of the fecal water can be accounted for by the sto
ol electrolyte composition: {2 [(Na+) + (K+)]} = [2 (39 + 96)] = 270. A villous
adenoma of the colon typically produces a secretory diarrhea. Lactose intoleranc
e, nontropical sprue, and excessive use of milk of magnesia produce osmotic diar
rheas with osmotic "gaps" caused by lactose,

carbohydrates, and magnesium, respectively. Pancreatic insufficiency causes stea


torrhea, not watery diarrhea.
A 60-year-old white female is scheduled to have a total abdominal hysterectomy.
She is currently in good health, but the general surgeon is concerned because th
e patient had a pulmonary embolus 10 years ago.Which one of the following is mos
t effective for prevention of another embolus? a.No prophylaxis necessary b.Impe
dance plethysmography, 36 and 72 hours after surgery c.Aspirin prophylaxis d.Ful
l heparinization after surgery e.Subcutaneous heparin prophylaxis e- This patien
t is considered at high risk for a venous thromboembolism because of general sur
gery, age greater than 40, and previous history of a pulmonary embolus. In numer
ous clinical trials, heparin, 5000 U subcutaneously 2 hours prior to surgery, fo
llowed by 5000 U subcutaneously every 8 to 12 hours until the patient is ambulat
ory, has statistically reduced the incidence of deep vein thrombosis. Full hepar
in therapy is not necessary. Aspirin is not effective, and impedance plethysmogr
aphy would not prevent thrombosis.
An obviously intoxicated 50-year-old white male is brought to the emergency depa
rtment after the car he was driving hit a telephone pole. He has a fracture of t
he femur, and is confused and uncooperative. He is hemodynamically unstable. Ini
tial physical examination of his abdomen does not indicate significant intra-abd
ominal injury. Which one of the following would be best for determining whether
laparotomy is needed? a.Peritoneal lavage b.Contrast duodenography c.Ultrasonogr
aphy of the abdomen d.An MRI scan of the abdomen e.A CT scan of the abdomen a- P
hysical examination of the abdomen is often unreliable for detecting significant
intraabdominal injury, especially in the head-injured or intoxicated patient. I
n a hemodynamically unstable patient with a high-risk mechanism of injury and al
tered mental status, peritoneal lavage is the quickest, most reliable modality t
o determine whether there is a concomitant intra-abdominal injury requiring lapa
rotomy. Computed tomography of the abdomen and contrast duodenography may comple
ment lavage in stable patients with negative or equivocal lavage results, but in
an unstable or uncooperative patient these studies are too time-consuming or re
quire ill-advised sedation. Ultrasonography may also complement lavage in select
ed patients, but its

usefulness is limited in the acute situation. Magnetic resonance imaging is extr


emely accurate for the anatomic definition of structural injury, but logistics l
imit its practical application in acute abdominal trauma.
A 67-year-old white male retired factory worker was found to have an abdominal a
ortic aneurysm on routine physical examination. A vascular surgeon has recommend
ed operative repair. The patient is a former smoker, with a 10-pack-year smoking
history. Three years ago, while under the stress of a forced retirement, he had
angina which responded well to sublingual nitroglycerin. He has had no angina f
or 1 year. Which one of the following conditions would be the most likely cause
of perioperative death in this patient? a.Pulmonary complications b.Peripheral e
mbolization c.Operative site infection d.Myocardial infarction e.Cerebral infarc
tion Answer is d. Myocardial infarction is the major cause of perioperative deat
h in patients undergoing surgery for abdominal aortic aneurysm. This is especial
ly true of patients with a history of known coronary artery disease or with EKG
abnormalities, and it is true even if the coronary artery disease is stable. Cer
ebral infarction occurs infrequently, even though cerebral vascular disease is c
ommonly present in this patient group. Operative site infection, peripheral embo
lization, and pulmonary complications are all significant postoperative complica
tions which may cause death, but they are less frequent causes of death than myo
cardial infarction.
Which one of the following is true of a pregnant patient who has diabetes mellit
us? a. Adjustments in hypoglycemic medications are best made by following urine
glucose readings b. Oral hypoglycemic agents are useful during pregnancy in pati
ents with mild diabetes mellitus c. A precise knowledge of fetal age is importan
t to a successful outcome for the fetus d. Because the fetal pancreas helps cont
rol the diabetes, ketoacidosis is less likely during pregnancy c- Ideally, the c
hild of an overtly diabetic woman should be delivered close to term. Precise kno
wledge of fetal age (by menstrual history, accurate measurements of uterine heig
ht during the second trimester, and confirmation by sonography) is very importan
t to a successful outcome for the fetus.

Severe growth retardation is diagnosed in the fetus of a 19 y/o unmarried white


female at 36 wks gestation. The Dx is based on biparietal diameter and there is
scant amniotic fluid.Which one of the following is the most appropriate manageme
nt? a. Perform serial L/S ratios until greater than 3.0, followed by prompt deli
very b. Induce labor, with careful fetal monitoring c. Perform an immediate C/S
d. Follow the mother weekly with serial ultrasounds e. Follow the mother weekly
with NST b- Appropriate management of the preterm infant who is severely growth
retarded depends on several factors. Generally those near term should be deliver
ed promptly. By the time that growth retardation has become severe, the fetus is
usually mature enough to survive if delivered promptly. However, the fetus must
be monitored carefully during labor, with facilities for immediate cesarean sec
tion if there is deterioration, and the neonate must receive excellent neonatal
care beginning immediately after delivery.
A 53 y/o woman presents with a long Hx of dyspepsia, fullness, and belching. An
endoscopy shows severe antral erythema and a small hiatal hernia, but no ulcer.
Because of her long history of dyspepsia, a serum gastrin test and serum IgG tes
t for Helicobacter pylori are done. The serum gastrin test result is 1800 pg/mL
and the test for H. pylori is negative. The next test that should be done in thi
s patient is: a- push enteroscopy b- secretin stimulation test c- gastric acid l
evel messurement d- pancreas angiography e- endoscopic US answer is c. the findi
ng of a high serum gastrin concentration is not infrequently a reason for referr
al for tests to diagnose Zollinger-Ellison syndrome, she does not have any of th
e features of the ZE, (recurrent multiple peptic ulcers and/or diarrhea). The fi
rst test that should be done is to assess whether the patient has hyperchlorhydr
ia or hypochlorhydria by inserting a NGT and measuring basal acid secretion. How
ever, some of the highest serum gastrin concentrations are found in patients wit
h achlorhydria or hypochlorhydria due to gastric atrophy (for example, in pernic
ious anemia or after the prolonged use of antisecretory drugs, particularly prot
on-pump inhibitors).

A 42 y/o female with Hx of 3 spontaneous abortions presented with pain in the le


ft leg, which later on Dx of DVT was confirmed. Which of the following findings
is most likely in the laboratory results of this patient? A - Polycytemia B - Th
rombocytopenia C - Low white blood cell count D - Hyponatremia E Hyperkalemia bIn this patient with confirmed DVT and remarkable Hx of the spontaneous abortio
ns, is highly suggestive of antiphospholipid antibody syndrome (antibodies direc
ted against either phospholipids or plasma proteins bound to anionic phospholipi
ds). Most common symptoms are venous and arterial thrombosis, recurrent fetal lo
sses, and thrombocytopenia. Other possible findings include livedo reticularis,
migraine headaches, Raynauds disease, hemolytic anemia, neurologic dysfunction, r
enal disease, pulmonary hypertension, avascular necrosis, and adrenal insufficie
ncy.
A 53 y/o male smoker presented with a 2 wk Hx of expectorating sputum streaked w
ith a blood. He denies any fever or chills or increases in the intensity of his
cough. P/E and CXR are unremarkable. Which of the following is an appropriate ne
xt step in the diagnostic work-up of this patient? A - Fiberoptic bronchoscopy B
- MRI of the chest C - High resolution CT of the chest ( HRCT ) D - A and B E A and C Answer is E. Fiberoptic bronchoscopy and HRCT are complementary to each
other. HRCT can demonstrate all tumors seen on bronchoscopy as well as several
which are beyond bronchoscopic range. On the other hand, HRCT could not detect b
ronchitis or subtle mucosal abnormalities that could be seen on bronchoscopy. Al
so, HRCT is particularly useful in diagnosing bronchiectasis and aspergillomas,
while bronchoscopy is diagnostic of bronchitis and mucosal lesions such as Kapos
is sarcoma. The patient in question is at high risk for pulmonary carcinoma; as o
f today, the procedures are considered complementary in this setting.
A patient with cytomegalovirus retinitis has been treated with IVganciclovir for
the last 4 months. On the most recent lab findings a sudden drop in the plt cou
nt was noted (20,000 mm3). Which of the following is the most appropriate action
in this situation?

A - Discontinuation of the therapy B - Exchange of ganciclovir with intravenous


foscarnet C - Exchange of ganciclovir with acyclovir D - Exchange of ganciclovir
with valcyclovir E - Continuation of ganciclovir therapy Answer is B. The major
drugs that are used for treatment of CMV retinitis are IVganciclovir and foscar
net, oral ganciclovir, intraocular ganciclovir, and IV cidofovir. Ganciclovir an
d foscarnet have equivalent efficacy against the retinitis. Major side effects o
f the ganciclovir are neutropenia and thrombocytopenia (limiting use in up to 16
% of patients). Ganciclovir should not be given with absolute neutropenia of les
s than 500 mm3 and thrombocytopenia 25, 000 mm3.
A 76 y/o man is hospitalized with stroke in the area of the right middle cerebra
l artery. He had a paroxysm of cough immediately after attempt to eat.On P/E he
is alert and oriented to time, place, and person. He has severe dysarthria but n
o signs of aphasia. He has facial asymmetry due to Lt sided facial droop, but hi
s gag reflex is intact. Which of the following is the most appropriate way to pr
ovide nutrition to this patient? A - Placement of a percutaneous gastrostomy tub
e B IV alimentation C - Feeding through a NGT D - Oral feeding supervised by a n
urse and suctioning as needed E - Clear liquid diet with advanced diet as soon a
s possible depending on patients clinical status Answer is B. After a stroke abou
t 25-45% of all patients develop dysphagia. The main problem that stems from dys
phagia is aspiration pneumonia, which, if it develops, greatly complicates the c
linical course and contributes to mortality. The patient in question had an atta
ck of cough after an attempt to eat. This is a common sign of dysphagia. Physica
l examination of this patient revealed several findings that suggest dysphagia (
facial nerve paresis and dysarthria). It is a common misconception that presence
or absence of a gag reflex correlates with the risk of aspiration. This is not
true. More important in the assessment of the aspiration risk are speech articul
ation, ability to swallow, and tongue movement.
A 16 y/o male in good health presents to you for a school physical. His family h
x reveals that his father died of colon cancer at age 37. Your physical exam rev
eals a healthy young man with several lipomas on his back and legs and a nodule
on his jaw. You should: a. Suggest colonoscopy at age 30. b. Suggest colonoscopy
now. c. Suggest flexible sigmoidoscopy and barium enema at age 30.

d. Suggest flexible sigmoidoscopy and barium enema now. e. Biopsy his jaw lesion
. Answer is D. case of Gardners Syndrome.
An elderly male complains of severe muscle weakness in his thigh muscles and pro
ximal arm muscles, although this weakness is mild. He states that his weakness i
s worse in the morning immediately after getting out of bed and improves during
the day. On P/E, it is apparent that muscle strength increases with repetition o
f the grip strength test and later diminishes. Which of the following tests shou
ld be included in the workup of this patient? A - CXR B - Tensilon test C - Abdo
minal CT D - Colonoscopy E Cystoscopy Answer is A. Symptoms in this man are cons
istent with Eaton-Lambert syndrome. This syndrome shares the same pathologic sit
e with myasthenia gravis (the neuromuscular junction) and has a similar path phy
siology (an autoimmune disease). Eaton-Lambert syndrome is usually associated wi
th malignancy. Up to 70% of these patients have associated small cell lung cance
r, and this disease must be ruled out in every patient with presenting symptoms
of Eaton-Lambert syndrome. Clinical presentation of the Eaton-Lambert syndrome i
ncludes weakness that is typically seen early in the hip girdle, making it diffi
cult for the patient to rise from a chair or to climb the stairs. Less dramatic
is shoulder girdle weakness. Involvement of the bulbar muscles or diplopia is ra
re, but ptosis is frequently seen. Symptoms are also likely to be more prominent
in the morning; autonomic dysfunction may cause erectile dysfunction and dry mo
uth. Differentiation from myasthenia gravis may be hard since patients with Eato
n-Lambert syndrome have a positive AChR-Ab test in 13% of cases.
A 50 y/o man presents to the ER complaining of 6 hours of severe diarrhea. He ha
s had eight to ten liquid stools, without blood. He has diffuse cramping abdomin
al pain alleviated by each bowel movement, but he denies tenesmus. He has mild u
ntreated hypertension but no other medical problems. He denies recent travel. Ot
her family members have not been ill. Vital signs: PR= 94/min, BP= 148/96 mm Hg,
no changes with standing, T= 37 C (98.6 F). The P/E is normal. The best initial a
pproach to this patient is: (A) Obtain stool specimens for culture, examination
for fecal leukocytes, and examination for ova and parasites

(B) Empiric antidiarrheal therapy (C) Empiric antibiotic therapy with trimethopr
imsulfamethoxazole or ciprofloxacin (D) Reassurance and oral fluids (E) Flexible
sigmoidoscopy with mucosal biopsy Answer is D. This patient has an acute diarrhe
al illness without evidence of severe inflammation, dehydration, or other risk f
actors. The great majority of acute diarrheal illnesses are infectious (viral or
bacterial) and self-limited. Oral hydration and reassurance are the most approp
riate care for such patients. If the diarrhea persists or if the patient develop
s a more toxic clinical picture, additional diagnostic tests would be indicated.
At the time of presentation, stool studies are not helpful. Empiric therapy wit
h antibiotics is not indicated. Flexible sigmoidoscopy is indicated only in pers
istent or inflammatory diarrhea.
A 9 y/o girl is brought to the clinic because she has felt "sick" and has been u
nable to go to school for the past 2 days. She complains of a headache, congesti
on, rhinorrhea, and double vision. Her PMH is remarkable for recurrent otitis me
dia 2 years ago for which she eventually received bilateral myringotomy tubes. S
he lives at home with her mother and grandmother who are both cigarette smokers.
T=38.8 C (101.8 F), PR=120/min, RR=20/min, and visual acuity is 20/20 in both e
yes. P/E shows tympanic membranes with evidence of previous surgery, but are oth
erwise normal, an erythematous oropharnyx with exudation, and slight exophthalmo
s of the left eye. On the left, the ocular examination also demonstrates periorb
ital edema, injection of the conjunctiva, trace restriction of extraocular movem
ents, and an afferent pupillary defect. The right eye is normal. The rest of her
physical exam is unremarkable. Whats the best next step in Mx? A. A CT of the he
ad B. Gentamycin eye drops treat most cases, but patients need close observation
C. Hospitalization and IV antibiotics are indicated D. Oral antibiotics are the
first line of treatment The correct answer is C. This patient has orbital cellu
litis. Patients present with lethargy, fevers, signs and symptoms consistent wit
h sinusitis, proptosis of the globe, restriction of extraocular movements, perio
rbital edema, injected conjunctiva, and afferent pupillary defects. Hospitalizat
ion and IV therapy should be started at once as this condition is potentially fa
tal, especially if the cavernous sinus becomes involved.

A 37 y/o woman with a history of bipolar disorder is brought to the ER by ambula


nce obtunded and ataxic. The patient has not been hospitalized in over 5 years a
nd has been stably maintained on outpatient therapy with a regimen of lithium ca
rbonate 600 mg by mouth twice a day. While you are examining her, she has a toni
c-clonic seizure lasting approximately 30 seconds. As part of the laboratory wor
k-up, the patient s lithium level is found to be 4.2 mEq/L. After basic airway a
nd cardiovascular support, the most appropriate next step in this patient s mana
gement is to: A. administer a loading dose of phenytoin B. get a renal consultat
ion for emergent dialysis C. order an MRI of the patient s head D. provide conse
rvative management including fluid and electrolyte replacement E. send liver fun
ction tests The correct answer is B. The patient presents to the ED with potenti
ally life-threatening lithium toxicity. The treatment of choice for lithium leve
ls greater than 4.0 mEq/L is hemodialysis.
A 16 y/o girl is brought to see you by her mother, who is a nurse in your office
. The patient is hoping to play for the school volleyball team and needs a clear
ance form signed by a doctor. Her past medical history includes exercise-induced
asthma and allergic rhinitis for which she takes albuterol and loratadine. Duri
ng the interview with the patient, while you are speaking with her alone, she re
veals that she has been sexually active with her boyfriend for the last 2 years.
She has not yet discussed this fact with her parents. Her physical examination,
including pelvic exam, is normal. At this time the patient should be screened f
or : A. chlamydia B. gonorrhea C. hepatitis B D. HIV E. herpes simplex F. syphil
is

The correct answer is A. Routine screening for chlamydia in all sexually active
women aged 25 and younger, as well as in asymptomatic women older than 25 who ar
e at high risk is recommended.
An 18 y/o boy is brought to the ED by ambulance. He and his friends were ice-ska
ting on a frozen pond when the ice broke and the young man became submersed in c
old water. His friends admit to drinking alcohol prior to the incident. The pati
ent s rectal T=33 C (91.4 F), PR= 57/min & weak, BP= 80/30 mm Hg, and RR= 7/min.
P/E reveals a comatose young man. An EKG is most likely to show A. delta wave B
. diffuse broadening of T waves C. prominent U waves D. shortened QT intervals E
. upward deflection following the S wave The correct answer is E. Hypothermia pr
olongs repolarization causing an upward deflection following the S wave (distinc
tive convex elevation of the J point), which is usually called an Osborn wave (o
r J wave of Osborn).
A 71 y/o woman is brought to the physician by her distressed daughter. The daugh
ter relates that, 3 days ago, her mother began to complain of RUQ abdominal pain
and then she complained of chills, nausea, and some vomiting. On P/E reveals an
obtunded, hypotensive, and obviously very sick elderly woman. She has impressiv
e pain to deep palpation in theRUQ, along with muscle guarding and rebound. T= 4
0 C (104 F). Lab results : WBC= 22,000/mm3 with multiple immature forms, bilirub
in= 5 mg/dL and Alk Ph= 840 U/L. serum amylase= normal. An emergency sonogram sh
ows multiple stones in the gallbladder, normal thickness of the gallbladder wall
without pericholecystic fluid, dilated intrahepatic ducts, and common duct with
a diameter of 2.1 cm. The sonographer cannot identify stones in the common duct
. In addition to IV fluids and antibiotics, which of the following is the most a
ppropriate next step in management? a- Elective cholecystectomy b- Emergency dec
ompression of the common duct c- Emergency cholecystectomy

d- Emergency surgical exploration of the common duct e- Emergency transhepatic c


holecystostomy The correct answer is B. The Dx is acute ascending cholangitis. T
his deadly disease is seen in patients with longstanding gallstones who get one
or more stones in the common duct, where they produce partial obstruction that a
llows ascending infection. The chills, very high fever, and extremely elevated a
lkaline phosphatase are diagnostic. The key component of therapy is immediate de
compression of the common duct, which is full of pus. How it is achieved is less
important. ERCP is usually the first choice, but it can be done by PTC or by op
en surgery.
33 y/o woman G3P2, with Hx of two preterm deliveries, comes at 32 wks gestation
with contraction every 4 min & 3 cm cervical dilation. A tocolytic drug administ
ered which raised her serum Glc from 95 to 170. What was the pharmacologic agent
? a- Terbutaline b- Magnesium sulfate c- Indomethacine d- Nifedipine e- Meperid
ine hydrochloride The answer is a- Terbutaline. Its a beta 2 agonist which relaxe
s the smooth muscle of uterus and its SE is raising blood Glc through glycogenoly
sis.
An febrile 53 y/o woman developes hypotension, tacchycardia & oliguria 24 hrs af
ter abdominal cholecystectomy. Her skin is cold & clammy. Which of the ffg is mo
st responsible for her symptoms? a- Gram (-) sepsis b- Hemoperitoneum c- Acute M
I d- Pulmonary embolus e- Pneumothorax Answer is b- hemoperitoneum. She shows cl
assic symptoms of hypovulemic shock.
A 6 month old child falls and hits his head. Which of the following signs would
be the MOST indicative of serious neurological injury? a- The parent states the
child was pale and sweaty for a few minutes after the fall b- A single post-fall
episode of emesis c- Lethargy immediately after the head injury

d- Hypotension e-A single grand mal seizure immediately after the fall Infrequen
tly, infants become hypotensive from blood loss into either the subgaleal or epi
dural space after head trauma. Hypovolemia can occur because of open cranial sut
ures and fontanelles. Transient paleness, lethargy, diaphoresis, and emesis are
common after minor head trauma and do not necessarily signify significant neurol
ogical injury. Persistence of any of these signs or symptoms, or change in menta
l status is concerning. Seizures may occur shortly after head injury and are usu
ally self-limited. However, about 50 percent of patients with posttraumatic seiz
ures have positive findings on head computed tomography (CT). Children with two
or more seizures or a GCS < 8 should be strongly considered for anticonvulsant t
herapy.
Which of the following drugs is NOT associated with acute pancreatitis? (A) Hepa
rin (B) Furosemide (C) Rifampin (D) Salicylates (E) Warfarin The answer is A. Dr
ugs and toxins are major causes of acute pancreatitis. Some of the medications a
ssociated with the occurrence of acute pancreatitis are oral contraceptives, est
rogens, phenformin, glucocorticoids, rifampin, tetracycline, isoniazid, thiazide
diuretics, furosemide, salicylates, indomethacin, calcium, warfarin, and acetam
inophen. Other etiologic factors contributing to the development of pancreatitis
include infection, collagen vascular disease, metabolic disturbances, and traum
a.
An 11 y/o boy is rushed into the ED following a MVA. The patient was a restraine
d, front seat passenger, when an oncoming car hit the passenger side of the car.
The boy denies loss of consciousness, but in the hospital he complains of pain
over his right side. His BP=118/59, PR=139/min, and RR=24/min. On P/E, he has de
creased breath sounds over the right base, and there is ecchymosis over the righ
t flank. His abdomen is soft with tenderness in the RUQ. Appropriate Mx of his a
irway with neck stabilization is provided and he is resuscitated appropriately.
Imaging studies of his neck are negative and a CXR does not show pneumothorax or
rib fractures. Dipstick of spontaneously voided urine is positive for blood. U/
A confirms the presence of 50 RBC/hpf. In regards to his hematuria the next most
appropriate course of action is to: A. begin empiric antibiotic therapy B. obta
in a CT scan of abdomen and pelvis C. order a renal/bladder ultrasound

D. perform a retrograde urethrogram E. place a Foley catheter F. repeat the urin


alysis G. schedule an outpatient intravenous pyelogram (IVP) The correct answer
is B. obtain a CT scan of abdomen and pelvis. The management of hematuria associ
ated with trauma differs in adults and children. In the adult population, imagin
g is performed only in those patients with gross hematuria or microscopic hematu
ria plus hypotension. This differs from the pediatric patient. In children, any
degree of hematuria (gross or microscopic) should be investigated with imaging s
tudies. One reason for this discrepancy is that large amounts of catecholamines
released in injured children may sustain blood pressure in the face of hypovolem
ia. A CT scan is the most useful imaging modality in this setting. A CT scan is
noninvasive, accurate and fast, and it can help in assessing the size and extent
of retroperitoneal hematomas and renal parenchymal trauma. Not only does this c
hild have microscopic hematuria (an indication by itself to perform imaging stud
ies), but he also has signs, (flank ecchymosis and tenderness), that raise the s
uspicion of renal injury. High suspicion for renal injury (i.e., rib fracture, f
lank contusion, deceleration injury) is another indication for perform imaging s
tudies.
*** A 45 y/o woman presents with acute pancreatitis with a serum amylase concent
ration
of 2010 U/L, T=37.2 C (99.0 F), the leukocyte count is 12,000/mL; plasma Glc=250 m
g/dL; and serum ca=8.4 mg/dL. She recovers after 1 wk of supportive therapy. No
obvious cause for the pancreatitis is found; ultrasonography is normal, and the
fasting serum triglycerides are 350 mg/dL. The best course of action to diagnose
the cause of the pancreatitis is: (A) ERCP (B) No further diagnostic testing un
til another attack occurs (C) CT scan with contrast (D) Measurement of serum CA
19-9 (E) MRI of the pancreas B- No further diagnostic testing until another atta
ck occurs The best course of action in a patient with a first attack of relative
ly mild pancreatitis even with a high serum amylase concentration is to see whet
her further attacks occur before continuing invasive investigations. The presump
tion is either that the patient has passed a small single gallstone or that the
cause has not yet been identified, that is, it is

idiopathic pancreatitis. The latter is often a once-only disease, and therefore no


further investigations are warranted at this time. An ERCP and CT scan or MRI w
ould be indicated after recurrent mild attacks or two severe attacks to try to e
stablish a diagnosis, particularly common duct stones or pancreas divisum. Measu
rement of CA 19-9 is not indicated. Although a full lipid workup in this patient
would be good medicine and should be done in any case (and may also be an indir
ect evidence of gallstones), it is unlikely to be the cause of pancreatitis. Hyp
erlipidemic pancreatitis is unusual if lipid levels are less than 600 mg/dL. Aft
er an initial severe attack, some authorities would do a repeat ultrasound withi
n 3 to 6 months just to exclude gallstones.
***1- An African- American couple with no known family hx of sickle cell dis., w
ant to know what the chance is that their child have sickle cell dis. To obtain
a more accurate assessment, RBC electrophoresis of the couple was done. The male
has no HbS, but the female has a clear HbS band. Knowing that the carrier rate
among African-Americans is approximately 8%, or 1 in 12, the chance that their c
hild will have sickle cell dis. Is most likely which of the following? a- 0 b- 1
/144 c- 1/288 d- 1/576 e- 1/1936
2- If there was no information about the parent electrophoresis, what would be t
he chance of the child having the dis.? a- 0 b- 1/144 c- 1/288 d- 1/576 e- 1/193
6 1- a- 0, 2- d- 1/576 Px with sickle cell dis., are homozygous for the HbS. Bec
ause only one potential parent has the HbS allel, the only way their child could
inherit two HbS genes is if the fathers sperm underwent a new mutation, which is
an extremely unlikely event. Therefore, their child does not have a chance of i
nheriting the dis. Howerever, theres a chance of that their child would be a carr
ier, having only the sickle cell trait. With no information, the odds that two c
arriers will meet is 1/12 x 1/12= 1/144, and the odds that two carriers will hav
e an infected child is , so, the odds that two Africaamericans will have a child
with dis. is 1/144 x = 1/ 576

***A 28 y/o woman, G4P4, at 28 wks gestation, presents for prenatal visit. She s
ays that she has not felt the baby move for 2 days. She has a Hx of HTN, which s
hes being treated with alfa methyl dopa, BID. On P/E, BP= 145/85, fundal height a
t 30 cm, and babys in transverse position. No FHR was detected with Doppler feoto
scope. Whats the next step of Mx ? a- Perform a NST b- Perform an amniocentesis c
- Obtain real-time US for cardiac motion d- Obtain a maternal abd. x-ray for fet
al assessment e- Obtain a quantitative beta HCG f- Observation and check up in 2
wks Answer is c- Obtain real-time US for cardiac motion Its the method of choice
for fetal death.
*** A 9 y/o boy presents with confusion & decreased school performance, he soon
developes a spastic gait with dysarthria, dysphagia & visual loss. MRI shows mas
sive white matter demyelination in the post. hemisphere. Whats the most likely Dx
? a- MS b- Metachromic leukodystrophy c- Adrenoleukodystrophy d- Subacute sclero
sing panencephalitis e- CJD Answer is c- adrenoleukodystrophy which is an x-link
ed dis. that affects young males, usually begin at 7-8 yrs of age. Its characteri
zed by demyelination of the CNS, adrenal insufficiency, mental deterioration, ap
hasia, apraxia & dysarthria. 1/3 of patients have loss of vision also.
*** An elderly Px developes painful swelling of Rt. parotid gland 10 days after
cholecystectomy. This is most likely secondary to which of the ffg ? a- Staph. A
ureus b- Duct obstruction by a stone c- A viral infection d- Hemorrhage e- An im
munologic reaction Answer is a- staph aureus. The px most likely has acute surgi
cal parotitis ( sialadenitis ), which usually occur 1 wk post op in elderly pati
ents who have poor dental hygiene and have intubated. Staph aureus is the most c
ommon organism. Tx is surgical drainage & antibiotics.

Which of the ffg can confirm the Dx of gastroparesis? (A) Endoscopy (B) Upper ga
strointestinal series (C) Ultrasonography (D) Nuclear medicine scintigraphy (E)
Electrogastrography Answer is D- nuclear medicine scintigeraphy There are no spe
cific signs and symptoms of delayed gastric emptying, and when this disorder is
suspected, the diagnosis of gastroparesis can usually be made by various diagnos
tic methods. The presence of retained food in a dilated stomach at the time of u
pper gastrointestinal contrast studies or detected endoscopically is indicative
of underlying motor dysfunction when mechanical obstruction has been excluded. H
owever, some patients with decreased motility of the stomach may not exhibit ret
ained food. Ultrasonography has been used to determine the emptying rate for liq
uids and appears to be relatively accurate, but gastric scintigraphy of either s
olid or liquid test meals labelled with a radiopharmaceutical has proven to be t
he most accurate method of detecting underlying gastric motility abnormalities.
Gastric scintigraphy supplies a quantitative result and can be used to evaluate
the response to therapy. Electrogastrography is a research tool and its clinical
application is unclear.
A 53 y/o alcoholic man with known cirrhosis presents with recent onset of abdomi
nal swelling and fever. On P/E PR=92/min, BP=98/62 mm Hg, T=38.5 C (101.3 F), palm
ar erythema, spider angiomata, moderate abdominal swelling with bulging flanks,
and a liver span of 8 cm are noted. There is no evidence of asterixis. Lab resul
ts: ALT=55 U/L, AST=62 U/L, serum BR=1.3 mg/dl, serum total pr.=7.3 g/dl, serum
Alb=3.8 g/dl Paracentesis reveals 25 mL of straw-colored fluid. Ascitic fluid al
bumin is 2.9 g/dL; the fluid contains 1000 RBC and 960 leukocytes/microliter, wi
th 90% lymphocytes and 10% PMN. The fluid is negative for amylase, and Gram stai
n shows no organisms.The most appropriate next intervention is:

(A) Cefotaxime, 1 g every 6 hours intravenously (B) Laparoscopy with biopsy and
culture of peritoneal lesions (C) CT scan (D) Norfloxacin for prophylaxis of spo
ntaneous bacterial peritonitis (E) Exploratory laparotomy Answer is B- laparasco
py ewith Bx & culture of peritoneal lesions This patient has evidence of chronic
liver disease on physical examination. The recent onset of ascites is complicat
ed by fever and an elevated number of ascitic fluid leukocytes with a lymphocyte
predominance. The ascitic fluid lymphocytosis is typical of tuberculous periton
itis, peritoneal carcinomatosis, or chylous ascites. In spontaneous bacterial pe
ritonitis, the absolute polymorphonuclear cell count is >250/L and represents ove
r 70% of the total leukocytes. This patient is at increased risk for tuberculous
peritonitis because of the debilitating conditions of alcoholism and cirrhosis.
The most sensitive procedure for diagnosing suspected tuberculous peritonitis i
s laparoscopy with biopsy for culture and histologic examination of peritoneal l
esions. This does not require surgical laparotomy. Diagnosis to the underlying c
ause of fever and ascitic fluid lymphocytosis should take precedence over initia
tion of diuretic treatment. Cefotaxime therapy is appropriate treatment for spon
taneous bacterial peritonitis, which is usually caused by enteric gram-negative
organisms or streptococci. Aminoglycosides are to be avoided in these patients b
ecause of the risk for precipitating or exacerbating renal insufficiency. After
an initial episode of spontaneous bacterial peritonitis, prophylaxis with oral a
ntibiotics that provide selective intestinal decontamination, such as norfloxaci
n, ciprofloxin, or trimethoprim sulfamethoxazole, will considerably reduce the in
cidence of recurrent peritonitis. CT scan is insensitive for diagnosis of infect
ion and is most useful for detecting malignancy, which is not likely in this pat
ient. CMDT: Fever may suggest infected peritoneal fluid, including bacterial per
itonitis (spontaneous or secondary). In immigrants, immunocompromised hosts, or
severely malnourished alcoholics, tuberculous peritonitis should be considered.
The accuracy of the SAAG exceeds 95% in classifying ascites. It should be recogn
ized, however, that approximately 4% of patients have "mixed ascites," ie, under
lying cirrhosis with portal hypertension complicated by a second cause for ascit
es formation (such as malignancy or tuberculosis). Thus, a high SAAG is indicati
ve of portal hypertension but does not exclude concomitant malignancy. In Wester
n societies, half of patients have underlying cirrhosis and ascites from portal
hypertension. In such patients, a diagnosis of tuberculous peritonitis may go un
suspected

because symptoms are attributed to the underlying liver disease. A high index of
suspicion is required for prompt diagnosis and treatment. http://www.sma.org/sm
j1999/aprsmj99/ahmad.pdf
A LBW infant at 2 months of age is brought because he has not benn eating well.
On P/E, hes pale & tacchycardic, The lungs are clear and theres no hepatosplenomeg
aly. CBC shows Hgb= 6 mg/dl, which of the ffg is the most likely cause of anemia
in this infant? abcdeMegaloblastic Sickle cell dis. Anemia of prematurity Alfa
thalassemia Homozygous beta thalassemia
Answer is c- anemia of prematurity, which occurs in LBW infants 1-3 months after
birth. Hgb is below 7-10 gr/dl.
You are called to see a 29 y/o woman, who was admitted to the hospital 24 hrs ea
rlier because of a stiff neck and a temperature of 39.7 C (103.4 F). She is pres
umed to have meningitis, but the results of the CSF analysis are still pending.
An LP was performed when she arrived at the hospital and IV antibiotics were sta
rted. She is now complaining of a dull, frontal headache that is worse when she
is sitting upright and relieved by lying down. V/S: T=39.3 C (102.8 F), BP=110/7
0, PR= 75/min, RR=18/min. P/E shows nuchal rigidity and there is a palpable purp
uric rash on her lower extremities and trunk. Funduscopic examination is unremar
kable. The most appropriate next step is to A. administer sumatriptan, intramusc
ularly B. advise her to remain in a horizontal position C. ask her if she has be
en drinking red wine D. order a CT scan of the brain to look for intracranial he
morrhage E. order an MRI of the brain to look for an intracranial mass The corre
ct answer is B. advise her to remain in a horizontal position

This patient most likely has a LP headache, which often begins within 24 hours o
f the procedure. The headache is positional in nature, meaning that it is worse
when she is sitting upright and relieved by lying down. It is believed to be due
to a loss of cerebrospinal fluid during the lumbar puncture, causing decreases
in the cushioning of the brain. Patients should be told to remain horizontal aft
er the procedure to decrease the incidence of this complication. The treatment i
ncludes remaining in a horizontal position and possibly intravenous caffeine sod
ium benzoate. If this is not effective, an epidural blood patch should be perfor
med.
A 28 y/o man comes to the ED because of moderate Lt. shoulder pain that is worse
with abduction of the shoulder. He plays baseball occasionally with friends and
has noticed that the pain worsens when throwing the ball. He denies any Hx of t
rauma. On P/E weakness of the shoulder, most pronounced with abduction is noted.
A shoulder x-ray reveals no fractures or dislocations. The most appropriate nex
t step in management is to A. advise patient that he should rest and ice the sho
ulder B. order an emergent CT scan of the shoulder C. order an emergent MRI of t
he shoulder D. order an outpatient MRI of the shoulder E. prescribe nonsteroidal
antiinflammatory drugs The correct answer is D. order an outpatient MRI of the
shoulder Rotator cuff tears are chronic use injuries most common in throwing ath
letes. The clinical symptoms are weakness and instability of the shoulder. An xray may show a subacromial spurring, a high-riding shoulder, or calcific tendoni
tis. Diagnosis is made by MRI of the shoulder. This is a non-emergent diagnosis
and the imaging can be performed on an outpatient basis. Repair is made arthrosc
opically.
A 17 y/o girl comes to the clinic because of a "reddish bump" on her abdomen. Sh
e states that she has had it for as long as she can remember, but her current bo
yfriend is concerned that it may be "cancer". She has no other complaints and do
es not take any medications other than ocp for birth control. Her grandfather ha
s malignant melanoma of the eye and her mother has breast cancer. P/E shows a 0.
4cm pinkish-brown, raised lesion on the left periumbilical region, 3cm lateral t
o the umbilicus. A small amount of soft tissue is palpated beneath the lesion. I
t is directly on an imaginary line that you can trace along the body from the le
ft axilla to the left inguinal region. The remainder of the

physical examination is normal. In discussing this lesion with the patient, you
should inform her that : A. coal tar ointment and ultraviolet light therapy is a
n old but useful treatment B. she has a 70% chance of having a kidney and/or uri
nary tract defect C. the lesion needs to be removed immediately because it is pr
ecancerous D. this is most likely a congenital defect that can be removed for co
smetic purpasdfasdfasdfoses E. this needs to be removed immediately and she will
need radiation therapy The correct answer is D. This patient most likely has an
accessory nipple (polythelia) and extraglandular breast tissue (polymastia), wh
ich are present in approximately 2% to 5% of the population. They usually occur
along the "milk line," which is an embryologic line that extends from the axilla
to the inguinal region. Normal breasts form along this line and other areas of
breast tissue and accessory nipples can too. Patients often think that they are
"moles" and generally are only concerned about the cosmetic appearance. They can
be simply removed as an outpatient, and additional treatment is not usually nec
essary.
A 74 y/o woman comes to the ED because of a 5 hr history of Rt sided weakness. T
he sudden weakness came on while she was giving her husband a bath. She says tha
t she was sure that she was having a stroke because her husband has had 2 stroke
s in the past, but her symptoms resolved spontaneously before she arrived at the
hospital. She is very concerned about her risk for a stroke because her husband
is completely dependent upon her for care. She is able to ambulate normally and
has no residual functional loss. BP=110/80 mm Hg, PR=70/min. Funduscopic examin
ation is normal. A thorough neurologic examination is unremarkable. The most app
ropriate next step is to : A. advise her to take a daily aspirin B. auscultate t
he carotid arteries C. obtain an MRI of the head D. order a carotid sonogram wit
h Doppler E. schedule an elective carotid endarterectomy The correct answer is B
. This patient most likely had a transient ischemic attack, which is a neurologi
c event that lasts for less than 24 hours. It is often due to atherosclerotic

disease and therefore auscultation of the carotid arteries may demonstrate the p
resence of a bruit, which suggests atherosclerotic disease at the carotid bifurc
ation. TIAs from carotid disease may occur due to ischemia from stenosis or from
an embolization of small plaques from the ulceration of the plaque. The next st
ep in this case is to auscultate the carotid arteries because this is the least
invasive, cheapest, and quickest method to detect atherosclerotic disease.
A 92 y/o man is brought to the clinic by his son and daughter who tell you that
their father "has gone blind". Further investigation uncovers that his visual lo
ss has been a slowly progressive process with no ocular pain. His son and daught
er are concerned because the patient lives alone. The patient does not complain
of any difficulty seeing and is agitated that he has been brought to your office
. Examination reveals normal vital signs and a visual acuity of 20/400 in both e
yes. Pupil examination, extraocular movements, and confrontational visual fields
are all normal. A penlight exam of the eye shows a yellow-brown color to the le
ns in both eyes, but is otherwise unremarkable. Direct ophthalmoscopy is very di
fficult and a sharp view of the retina is not possible. The family should be adv
ised that: A. Age alone is a common cause of decreased vision and his level of v
ision is what you would expect for a patient his age B. Eye surgery is necessary
to prevent total blindness (no light perception) C. Glaucoma is the most likely
diagnosis and prompt ophthalmology consult is indicated D. He has macular degen
eration and low vision aides should be considered to assist with activities of d
aily living E. Tell the patient and family that as long as the patient is comfor
table with his vision no treatment is necessary The correct answer is E. This pa
tient has cataracts in both eyes. The typical clinical presentation of a patient
with cataracts is that of slowly progressive visual loss. Patients may also com
plain of difficulty with glare and bright lights. They often note decreased colo
r brightness and contrast sensitivity. On exam, a lens that has a cataract will
appear yellow-green to yellow-brown. A mature cataract will present as a white l
ens and visual acuity of hand motions or light perception. With advanced catarac
ts, the fundoscopic exam may be difficult or not possible. The treatment for cat
aracts depends on the level of visual deficit experienced by the patient. Approp
riate treatments range from observation/education, spectacles, and cataract surg
ery. If a patient with cataracts is comfortable with his current vision, then tr
eatment with glasses or surgery is not necessary. Referral to an

ophthalmologist may be appropriate for more in depth counseling based on the lev
el of concern of the patient.
A 3 y/o boy is brought to the ED by his parents because of a 24 hr Hx of intermi
ttent, generalized abdominal pain. The parents tell you that he complains of the
pain for 10minute episodes and during these times he refuses to walk, but then
he spontaneously returns to his normal activities. This occurred 8-9 times yeste
rday. Today the symptoms occurred more frequently and were associated with 3 epi
sodes of non-bloody, non-billous emesis so the parents brought him into the hosp
ital. There is no history of fever, constipation, or soiling. On examination the
patient appears tired and has mild diffuse abdominal pain. He has guaiac-positi
ve stool. PR=125/min. The study most likely to provide a diagnosis is : A. an ab
dominal x-ray B. a barium enema C. a CBC with differential D. a CT scan of the a
bdomen E. a LP The correct answer is B. This patient presents with a very common
complaint in the pediatric population, abdominal pain. The key to this case is
the quality and frequency of this abdominal pain. The pain was described as bein
g diffuse and intermittent with periods of resolution of the symptoms. This type
of pain pattern, along with emesis, the lethargy seen in the emergency departme
nt, and the guaiacpositive stools should raise red flags for the diagnosis of in
tussusception. In intussusception a segment of bowel (most commonly the distal i
leum into the cecum) telescopes into an adjacent segment causing obstruction. Th
is obstruction tends to resolve and recur causing thasdfadsfe intermittent abdom
inal pain. The barium enema is diagnostic and in many cases a curative procedure
as well and is therefore the study of choice in this case.
A 64 y/o farmer comes to the clinic with an injury to the foot, which happened a
t his farm 24 hrs ago. He reports that he was working on the farm, when he accid
entally stepped on a rusty nail, which penetrated deep into his foot. He took so
me analgesics and he was feeling well. His wife urged him to come to the clinic.
He denies any fever, chills, or

rigor. His past medical history is significant for non-insulin dependent diabete
s mellitus, which is well controlled with oral therapy. He has no past surgical
history. Examination of the foot reveals a deep penetrating wound in the sole of
the left foot. There is no associated erythema or induration around the foot. N
o foreign particles are noted along the edges of the foot, which is tender on pa
lpation. There is no motor or sensory loss. No active bleeding is noted in the p
enetrating wound. The patient tells you that he had 3 doses of tetanus toxoid in
jections when he was young. The last tetanus injection was 8 years ago for a sim
ilar episode. The most appropriate next step in management is to A. administer a
tetanus toxoid booster injection B. administer a tetanus toxoid booster injecti
on and human immunoglobulin C. provide human immunoglobulin D. provide no additi
onal therapy at this time E. surgically debride the wound The correct answer is
A. Tetanus prone wounds are any wounds that are over 6 hours old, deep, crushed
or penetrated, contaminated with soil, associated with compound fractures, parti
al or full thickness burns, or human or animal bites. Patients who received 3 do
ses of tetanus toxoid in the last 5 years do not need further therapy for a teta
nus prone wound or for a clean wound. Patients who received a tetanus toxoid bet
ween 5-10 years ago and have a tetanus prone wound need booster tetanus toxoid i
njections, whereas those with a clean wound would need no further therapy. When
the last dose of tetanus toxoid was more than 10 years ago, then both clean woun
ds and tetanus prone wounds need tetanus toxoid booster injections. In addition,
those with a tetanus prone wound, whose last dose was more than 10 years ago, n
eed to have human immunoglobulin (choice C) administered.
** A 60 y/o diabetic male is brought to ER by his wife for evaluation. His wife
reports that he had developed high fevers 2 days prior, which were attributed to
a viral infection and no medical attention was sought. His wife became concerne
d when the fever persisted despite the use of Tylenol and he became increasingly
lethargic. Vitals: HR=130, RR=24, BP=70/55, T=102F On examination, he was flush
ed, warm to the touch, confused, and diaphoretic. Mucus membranes appeared dry.
His neck was supple with negative Kernig and Brudzinski signs. His eyes were equ
al and reactive to light with no papilledema, and his lungs were clear to ascula
tion bilaterally with no evidence of rales, rhonchi, or wheezes. On cardiac exam
he was tachycardic with a normal S1 and S2. A flow murmur was heard. The abdomi
nal and neurologic exams were unremarkable. Examination of the dorsal aspect of
the right foot revealed foot cellulites. What would be the first appropriate ste
p of treatment in this man?

A. Immediate dose of IV cefazolin (Ancef) B. Fluid resuscitation with normal sal


ine C. Drainage and culture of wound site D. Obtain CBC, Chem 10, ABG, blood cul
tures, urine cultures, wound culture E. Stat CT of adsfasdfasdfhead Answer is B.
This patient is in septic shock given his BP of 70/55 and temperature of 102F.
In addition, patient demonstrates flushing, which indicates vasodilation, anothe
r characteristic of septic shock. This is most likely the result of bacteremia s
econdary to his foot cellulitis. In hemodynamically unstable patients, airway, b
reathing, circulation should be assessed and fluid resuscitation should be start
ed immediately. This should be quickly followed by drainage and culture of wound
site, urine culture, and bloodwork including: CBC, Chem10, ABG, blood cultures.
Furthermore, empiric antibiotics should be started intravenously. In this case,
the most common organisms would be Strepococcus and Staphylococcus, making cefa
zolin (Ancef) would be an appropriate choice. Mental status changes are often ev
aluated by CT scan. However, this patient s changes are most likely secondary to
his hemodynamic instability and should resolve with resolution of his septic sh
ock.
** A 25 y/o female is sdfsdfasdfbrought to ER after being involved in a MVA. She
is hypotensive and tachycardic. She has good breath sounds bilaterally. Fluid r
esuscitation is started immediately with Lactated Ringers solution, and blood is
sent for type and cross. She has minor scrapes and lacerations on her torso. Sh
e remains hypotensive even after 2L of Lactated Ringers solution and blood trans
fusion with packed red blood cells is begun. After infusion of 2 units, her bloo
d pressure stabilizes at 110/80 mm Hg. She is placed under observation. 3 hours
later she complains of chills, generalized pruritus and difficulty breathing. On
exam she is found to have a temperature of 101 F, BP of 110/80, HR 100, RR 20.
HEENT is normal. Neck is supple with no JVD. Auscultation of the lungs reveals c
rackles at the bases. Heart exam is normal. Abdomen is soft, nontender, nondiste
nded, with good bowel sounds, there is no hepatosplenomegaly. Extremities reveal
no edema, no cyanosis, and good range of motion. Neurologic exam is unremarkabl
e. Exsdfadsfdsfadsfadfs brought in by the paramedics after he was found confused
and wandering in the streets. On evaluation he is agitated, confused and combat
ive. He is diaphoretic, emanates a foul odor, and his clothes are filthy with ev
idence of emesis. He has slurred speech, is noted to be talking to people who ar
e not there, and swatting at invisible objects. Upon questioning he makes an inc
oherent comment about seeing bugs on the walls. He has a generalized tremor and
appears fearful of the staff. His vital signs show: BP 135/80, HR 102, RR 20, Te
mp 99.8 F. His mucus membranes are dry and skin turgor is normal. Chest and card
iac exams are normal. Abdominal exam is significant for slight hepatomegaly. Neu
rologic exam reveals intact cranial nerves, DTR s are symmetric bilaterally, but
he refuses to cooperate with the remainder of the neurologic exam. He is disori
ented to person, time and place. Which of the following changes are consistent w
ith this patient s presentation? ALT MCV [Mg+2] Glucose No No A. inc dec dec cha
nge change B. inc inc inc dec dec No C. inc inc inc dec change No No D. inc inc
inc change change No No No E. inc inc change change change AST
Answer is B. This patient is exhibiting signs and symptoms of delirium tremens (
DT). DT is organic psychosis, which is usually the result of acute alcoholic wit
hdrawal. It is characterized by mental confusion, tremors, visual hallucinations
, diaphoresis, and autonomic hyperactivity. It usually manifests within 25-72 ho
urs after the last drink, but, can occur as late as 7-10 days later. DT tends to
occur in individuals with previous withdrawal episodes. This patient is a chron
ic alcoholic and alcohol can cause liver damage. As a result he can be expected
to have elevated AST and ALT levels. Usually AST is two times more elevated than
ALT. Gluconeogenesis in the liver may be impaired resulting in hypoglycemia. In
addition, hypoglycemia, hypomagnesemia and an elevated MCV are commonly seen in
alcoholics, and are most likely the result of malnutrition.

** A 19 y/o college student with height 5 2" and weight 120lbs is brought to the
emergency room after she experienced two episodes of coffee ground emesis. Her
vital signs on arrival are normal. On physical exam she has poor eye contact and
a blunted affect. She is oriented to person, time and place. Her mucus membrane
s are moist, front teeth show loss of enamel. She has mildly enlarged parotid gl
ands bilaterally. Her breath has a rancid odor. Her chest and cardiac exam are n
ormal. She has mild epigastric tenderness, good bowel sounds, no distention, no
hepatosplenomegaly. Neurologic exam is unremarkable. A nasogastric tube is place
d which drains clear fluid after an initial 10ccs of coffee ground material. An
upper endoscopy is performed which demonstrates a linear mucosal tear at the gas
troesophageal junction and a diagnosis of a Mallory-Weiss tear is made. From you
r understanding of the patient s underlying condition, which of the following ac
id-base abnormalities would you expect? A. Hypochloremic metabolic alkalosis B.
Hypochloremic metabolic acidosis C. Hyperchloremic metabolic alkalosis D. Hyperc
hloremic metabolic acidosis E. No abnormalities are expected.
Answer is A On Hx & P/E , this patient has many concerning signs and symptoms. H
er wornappearing enamel, bilaterally enlarged parotid glands, rancid breath odor
, and poor eye contact point to an eating disorder, most likely bulimia nervosa,
given her normal height to weight ratio. In addition to her chronic disease, sh
e is acutely experiencing pain secondary to a Mallory-Weiss tear. A Mallory-Weis
s tear ia a mucosal tear at the gastroesophageal junction, which arises from eve
nts that suddenly raise transabdominal pressure. Examples include sudden lifting
, or as in this patient, retching or vomiting. Given this patient s history of r
etching and vomiting, one would expect her to have hypochloremic metabolic alkal
osis, as H+ and Cl- are lost during vomiting.
45 y/o male with atrial rate of 300/min & ventricular response in 2:1 ratio, has
been treated with an antiarrhythimc agent, which slowered the atrial rate to 20
0/min & increased the ventricular rate to 200/min. What was the pharmacologic ag
ent used? a- adenosine b- digoxin c- propranolol d- quinidine e- verapamil Answe
r is d- quinidine.

Quinidine slowers the


uction. So, an atrial
1 ratio, resulting in
uld be avoided by use
, c & e.

atrial rate due to its vagolytic action & increases AV cond


flutter at 200/min was conducted to the ventricles in a 1:
an increase in ventricular rate to 200/min. This effect co
of a drug that slows AV nodal conduction like choices a, b

Which of the ffg correctly identifies the sequence of cardinal movement of labor
that fetus undergoes prior to delivery? a- descent, ext. rotation, extension, i
nt. rotation, flexion b- descent, flexion, int rotation, extension, ext rotation
c- int rotation, flexion, descent, ext rotation, extension d- flexion, extensio
n, int rotation, descent, ext rotation e- extension, descent, int rotation, ext
rotation, flexion answer is b- descent, flexion, int rotation, extension, ext ro
tation
35 y/o woman, G3P2, at 39 wks gestation undergoes a vaginal delivery of a 3295 g
r male neonate. The placenta was not completely removed & after bleeding she und
erwent a total abdominal hysterectomy due to placenta accreta & receives 5 unite
s of packed RBC. Which of the ffg hormones are most likely to be affected by thi
s scenario? a- ACTH b- PRL c- TSH d- FSH e- ADH f- None of the above g- All the
above Answer is b- PRL due to developing sheehans syn.
A 26 y/o woman with recently diagnosed Hodgkin s disease is admitted to the ICU
after an overdose of lorazepam and alcohol. She is intubated and stable. Which o
f the ffg would be expected to help prevent the development of ventilator-associ
ated pneumonia in this patient? A. Keeping the patient in the semi-recumbent pos
ition B. Routine changes of the ventilator circuit C. Nasotracheal (as opposed t
o oral) intubation D. Prophylactic antibiotics E. Chest physiotherapy

Answer is A. Keeping the patient in the semi-recumbent position Ventilator-assoc


iated pneumonia accounts for more than 50% of all infections in the ICU. Handwas
hing, semi-recumbent positioning, kinetic (rotational) bed therapy, and drainage
of condensate from ventilator circuits are effective strategies for preventing
such infections. However, routine changes of the ventilator circuit and chest ph
ysiotherapy are ineffective. There is some evidence to support oral, as opposed
to nasal, intubation. Influenza and pneumococcal vaccination, which should be ad
dressed in all patients at the time of hospital discharge, would not affect the
development of ventilator-associated pneumonia.
In a patient with steatorrhea, which of the following test results is the most s
pecific for the diagnosis of chronic pancreatitis? A. Abnormal Schilling test th
at does not normalize with intrinsic factor B. A serum carotene value of 25 g/mL
C. A 72-hour fecal fat content of 30 g D. A serum trypsinogen value of 5 ng/mL E
. An abnormal 13C triolein breath test Answer is D. a serum trypsinogen of 5 ng/
ml The serum trypsinogen concentration, when low, is the only test in this group
that is specific for pancreatogenous steatorrhea. The other test results are as
sociated with, or diagnostic of, steatorrhea but do not identify specific cause
of steatorrhea. The Schilling test, if abnormal after intrinsic factor, can occu
r with ileal disease, bacterial overgrowth, or chronic pancreatitis. A low serum
carotene concentration, abnormal 72hour fecal fat test, or abnormal 13C triolei
n breath test are present with steatorrhea of any cause, for example, small bowe
l mucosal disease, bacterial overgrowth, chronic pancreatitis, and ileal resecti
on.
A 38 y/o farmer has acute onset of severe, progressive dyspnea associated with c
ough and weakness. These symptoms occurred 1 hour after the patient had worked i
n a silo that had been filled with fresh corn silage the previous day. Which of
the following is most likely to have caused his acute pulmonary decompensation?
A. Exposure to thermophilic actinomyces B. Viral pneumonia C. Pneumothorax D. Ni
trogen dioxide pneumonitis E. Allergic bronchopulmonary aspergillosis

Answer is D. Nitrogen dioxide pneumonitis Silo filler s disease results from inh
alation of oxides of nitrogen, including nitrogen dioxide, which tends to accumu
late at the top of tall storage silos. The oxides of nitrogen are generated soon
after the silo is filled with fresh silage that is subsequently used for animal
feed. Nitrogen dioxide, when dissolved in the aqueous film that lines the respi
ratory tract, becomes nitric acid and produces a chemicale., with handgrip) the
end systolic chamber size is increased, the degree of outflow obstruction decrea
sed, and the intensity of the murmur is decreased. The prominent PMI suggests a
left ventricle that has hypertrophied in response to the outflow obstruction. Th
e identification of IHSS is important since those with this form of cardiomyopat
hy are at increased risk for sudden death.
A 57 y/o man with schizoaffective disorder, whose symptoms were in remission unt
il 2 wks ago, is brought to the ED by his girlfriend. He will not talk with you
but the girlfriend tells you that he has a hx of "overdoses", and she is afraid
he has taken a lot of "his pills". He has been complaining of voices telling him
he "should be dead". He has not left the house in a month and has spent several
hours a day looking out the window for the "king and savior" to "come take him"
. His medications include haloperidol, valproic acid, and a small dose of amitri
ptyline for chronic pain related to nerve damage in his leg, which occurred in a
motor vehicle accident 10 years ago. He has no other medical problems. A chart
review reveals that he has no allergies and was diagnosed with schizoaffective d
isorder 30 years ago. Vitals: T=37.0 C (98.6 F), BP=110/70 mm Hg, PR=70/min, and
RR=26/min. He is a depressed appearing man with very poor eye contact. He smell
s of alcohol. He does not acknowledge you, but will answer some questions for hi
s girlfriend. He does admit to feeling that "life is not worth living" and feeli
ng "more religious than usual". He seems slightly drowsy and knows the date. His
physical examination is normal. The most appropriate next step in management is
to order A. acetaminophen and salicylate levels B. blood alcohol level

C. ECG D. urine toxicology for street drugs E. valproic acid level The correct a
nswer is C.ECG This patient may have taken several cardiotoxic medications. Halo
peridol and other antipsychotics increase the QT interval, as do the tricyclics.
In an overdose for either medication, he is at risk for sudden death from torsa
des.
A 14 y/o boy is brought to the office by his mother because of "bedwetting" epis
odes that have been occurring about twice a week for the past few months. The mo
ther says that she noticed this "problem" when she washed his pajamas, and he "r
efused" to talk about it when she tried to bring it up. She is hoping that he wi
ll talk to you. You ask her to leave the room so you can have some privacy with
her son. He starts the conversation by stating that "this is getting embarrassin
g" and he "doesn t understand what s going on." He says that he gets up and find
s his pajamas "wet and sticky." He denies any dysuria or frequency during the da
y, and denies any problems at school or at home. He is on the basketball team, s
ocializes with friends, and gets good grades. Physical examination is unremarkab
le and shows a pubic hair stage of Tanner IV and genital development Tanner stag
e III. The next best step is to A. advise him to stop drinking water at 8 pm and
urinate before bed B. begin a 3-day treatment regimen with trimethoprim-sulfame
thoxazole C. obtain a urine sample for U/A & culture and sensitivity D. order a
renal ultrasound E. reassure him that this is a completely normal part of pubert
y F. order CBC & electrolytes The correct answer is E. reassure him that this is
a completely normal part of puberty This adolescent boy is most likely having n
octurnal emissions ("wet dreams"), which are a normal part of puberty. A nocturn
al emission is when the penis becomes erect during sleep and ejaculates. If the
boy or his mother do not know about nocturnal emissions before they occur, they
may think that he urinated. He should be reassured that this is normal. It is al
so important to discuss it with his mother so she does not make him feel uncomfo
rtable about it in the future.
A 36 y/o woman comes to the ED because of a severe headache. She states that the
headache woke her up from sleep 6 hours ago, and was not relieved by aspirin or
acetaminophen. She also noticed that she has neck stiffness and that "it hurts"
during neck extension and flexion. She was recently diagnosed with Hodgkin s di
sease, but before that, she was usually "pretty healthy," except for a few UTI a
nd HTN. V/S: T=39.0 C (102.2 F), BP=130/80 mm Hg, PR=75/min, and RR=17/min. She
appears

lethargic. P/E shows nuchal rigidity, flank tenderness, and a mid-systolic click
. Funduscopic examination shows bilateral optic disc swelling. Whats the most app
ropriate first step to do: A. B/C & administer IV ceftriaxone B. order a CT scan
of the head C. order a MR angiogram of the head D. order an MRI of the head E.
perform a LP The correct answer is A. B/C& IV ceftriaxone This patient has the s
igns and symptoms that are suggestive of meningitis, but she also has papilledem
a (optic disc swelling), which means that an intracranial mass must be ruled out
before performing a lumbar puncture (to reduce the risk for brain herniation).
However, empiric antimicrobial therapy should be started before the neuroimaging
study so that clinical deterioration does not occur.
A 17 y/o girl is brought to the office by her mother because she has missed many
periods. The girl admits to binge eating and exercising in order to prevent weigh
t gain. She tells you that she is definitely not pregnant, because she has not h
ad any sexual relations in the past 11 months and thinks she is not getting her
menstrual period because of the excessive physical exercise she has been doing i
n the past several weeks. P/E is significant for bradycardia and significant wei
ght loss compared to the last year. A pregnancy test is negative.At this time wh
ats the first step? A. order amylase B. check BUN and creatinine C. order LFT D.
check serum K E. thyroid function tests The correct answer is D. check serum K B
ulimic patients frequently engage in compensatory behaviors to prevent weight ga
in. Those include self-induced vomiting, abuse of diuretics, laxatives, enemas,
or diet pills. The metabolic disorders frequently seen in these patients are, mo
stly hypokalemia and hypomagnesemia. Thyroid function tests should be done as a
part of regular workup of patients presenting with this clinical picture. It is
not, however, the first to be ordered.
45 y/o truck driver involved in MVA, resulted in closed hesd injury, presents to
ER, he was intubated at the field and on arrival hes oxygenated well with assist
ed ventilation, has normal BP & moderate tacchycardia. GCS = 7, pupils are equal
& slowly reactive. After stabilization, a head CT is done which shows small SAH
& Rt. frontal lobe contusion. Abd. CT is normal. The optimal Mx of this px ICP
is : a- fluid restriction, hyperventilation, IV steroids

bcdefluid restriction, hyperventilation & ventriculostomy fluid restriction & osmoti


c diuresis normovolemia, normocarbia & ventriculostomy craniectomy
answer is d- normovulemia, normocarbia & ventriculostomy The principle of Mx of
closed head injury is to maintain cerebral perfusion & oxygenetaion to prevent s
econdary brain damage. Remember this formula: CPP ( cerebral perfusion pressure
)= mean BP ICP, So normal CPP requires adequate circulating blood volume with ma
intanace of normovolemia. Early ventriculostomy is beneficial to permit controll
ed drainage of CSF to maintain normal CPP. Note: Fluid restriction, hyperventila
tion, hypercarbia ( -- > vasodialation & inc. ICP ) should be avoided.
A 70 y/o man with hx of HTN & mild CHF ( which was controlled with digoxin & diu
retics ) is admitted for an AAA repair. To faciliate perioperative Mx, a swan- g
anz cath. is inserted in the OR. During a first few hours post op hes noted to ha
ve BP= 140/70, HR=110, flat neck veins, pulmonary arterial wedge pressure of 9 m
m/hg & poor urine output. 1- The next best step of Mx is : a- IV furesmide b- Bo
lous IV crystalloid c- Dopamine infusion d- Nitroprusside infusion e- IV digoxin
Several hours after this intervention, BP=150/85, HR=90, neck veins are distend
ed, pulmonary arterial wedge pressure is 17 and urine output is still low. 2- At
this point, Mx should be: a- IV furesmide b- Bolous IV crystalloid c- Dopamine
infusion d- Nitroprusside infusion e- IV digoxin Answers are: 1- b, 2-d In the i
nitial post op period, px has a low pwp & poor urine output, renal perfusion is
compromised by hypovolemia with subsequent inadequate preload & decreased cardia
c output. So at this time IV fluid resuscitation is appropriate. After fluid bol
ous px developes extended neck veins & elevated pwp, indicating biventricular dy
sfunction with increased left end diastolic pr. & increased left ventricular end
systolic volume. CO is low & urine outpout is not improved, so with px hx all s
cenario is due to increased afterload which can be reduced by nitroprusside infu
sion.

70 y/o man presents with back pain & difficulty urination. On DRT, he has a hard
, irregularenlarged prostate. PSA is elevated & osteoblastic lesions of vertebra
l column & pelic bones are noticed. The Tx of choice is: a- radical prostatectom
y b- transurethral prostatectomy c- cytotoxic chemotherapy d- hormonal manipulat
ion e- radiotherapy answer is d- hormonal manipulation this elderly px has metas
tatic prostate cancer, so Mx would be tumor control for palliation of symptoms,
which is hormonal manipulation either with orchiectomy or exogenous estrogen the
rapy.
A 75 y/o woman is brought to ER from nursing home for jaundice & mental confusio
n. The nusrsing home notes states that she became less responsive & developed ja
undice over the last 2 wks. PMH is positive for HTN, DM & prior colon resection
for colon cancer at age 55. V/S : BP= 100/60, HR= 110, T= 101.5, P/E shows no re
sponse to verbal command but withdraws to pain, a mild jaundice with tenderness
in epigastrium & RUQ. Whats the most likely Dx? a- Hepatitis A b- Biliary strictu
re c- Choledochal cyst d- Liver metastasis e- Choledocholithiasis f- Cirrhosis g
- Pancreatitis Answer is e- choledocholithiasis. Common bile duct stones may be
the cause of acute bile duct obstruction without warning resulting in jaundice,
pain & sepsis. The sepis may manifest as fever, hypotestion & altered mental sta
tus. A 66 y/o woman who has previously been healthy undergoes emergency surgery
for a ruptured AAA. Intraoperatively she requires 8 units of packed red blood ce
lls to maintain her blood pressure and hematocrit. After surgery she is hemodyna
mically stable. On the third post-op day she appears jaundiced, but abdominal ex
amination is unremarkable and she is afebrile. Lab results: Total serum BR=8.3 m
g/dL (direct= 6.3 mg/dL) Serum Alk. Ph.= 360 U/L Serum AST = 51 U/mL

The most likely explanation for the woman s jaundice is A: a stone in the common
bile duct B: halothane hepatitis C: posttransfusion hepatitis D: acute hepatic
infarct E: benign intrahepatic cholestasis The answer is E- Benign intrahepatic
cholestasis Benign post-op intrahepatic cholestasis can develop as a consequence
of major surgery for a catastrophic event in which hypotension, extensive blood
loss into tissues, and massive blood replacement are notable. Factors contribut
ing to jaundice include the pigment load from transfusions, decreased liver func
tion resulting from hypotension, and decreased renal bilirubin excretion caused
by tubular necrosis. Jaundice becomes evident on the second or third postoperati
ve day, with bilirubin levels (mainly levels of conjugated bilirubin) peaking by
the tenth day. Serum Alk. Ph. concentration may be elevated up to tenfold, but
AST level is only mildly elevated. Hepatitis, choledocholithiasis, and hepatic i
nfarct are unlikely diagnoses in the absence of abdominal tenderness, fever, or
a significant rise in AST levels. The incubation period of posttransfusion hepat
itis is 7 weeks, making this diagnosis unlikely.
A 6 y/o boy is brought by his mother because of a "red rash" that she noticed to
day. She says that 3 days ago he had a cough, runny nose, and fever that respond
ed to ibuprofen. T=37 C (98.6 F) and he has a normal P/E with the exception of a
n erythematous, blanching macular rash on his legs. You diagnose him with a vira
l exanthem and advise the mother to encourage the child to drink liquids and to
use ibuprofen as needed for fever. One week later, the mother brings the child b
ack to the office and reports that the rash has "changed", he has developed coli
cky abdominal pain several times per day, and he is complaining of left knee pai
n. V/S: T=37.2 C (99 F), BP=100/65 mm Hg, PR=100/min, and RR=15/min. P/E reveals
a well-appearing child with palpable purpura of both lower extremities, normal
neck examination, clear lungs, and a soft, non-tender abdomen. His left knee is
painful on flexion, but it is not erythematous or warm, and there does not seem
to be an effusion. His gait is normal. The most appropriate study at this time i
s : A. arthrocentesis B. colonoscopy C. cultures of blood, urine, and cerebrospi
nal fluid D. echocardiography E. urinalysis

The correct answer is E. This patient most likely has Henoch-Schonlein purpura (
HSP), a smallvessel vasculitis seen most commonly in children between the ages o
f 2 and 8. The child s preceding upper respiratory tract infection, low-grade fe
ver, and arthralgias are all common elements of this disease. The typical rash o
f HSP is an evanescent, erythematous, macular rash on the lower extremities that
progresses over the course of days to petechiae and palpable purpura. These cha
nge in color from red to purple to brown before eventually fading, normally over
the course of weeks. HSP is an IgA-mediated autoimmune vasculitis, which can ca
use tissue damage as a result of immune complex formation. Deposition of these i
mmune complexes in the kidneys can lead to nephritis, which is the leading cause
of permanent sequelae from HSP. End-stage renal disease is an uncommon but poss
ible outcome. It is important to perform frequent urinalyses for early detection
of kidney involvement.
A 5 y/o boy is admitted to the hospital because of his increasing irritability a
nd fever. The mother reports that the child has been having upper respiratory sy
mptoms for the past week. In the last few days, the child has been constantly ru
bbing his left ear and has been increasingly irritable. The patient has had mult
iple ear infections in the past, that were treated with oral antibiotics. Follow
ing recurrent episodes of these ear infections, the child was advised to take a
prophylactic, single dose of amoxicillin at bedtime. The child has been taking t
hese antibiotics regularly for the past 3 months. On examination, the child is f
ound to have a fever of 38.8 C (101.8 F). Examination of the right ear does not
reveal any abnormalities. The examination of the left ear is uncomfortable, but
the external auditory meatus appears normal. The tympanic membrane is examined a
fter the removal of the cerumen and is noted to be hyperemic, bulging with indis
tinct anatomical landmarks. Light reflex is diminished and there is limited mobi
lity on pneumatic insufflations. Some amount of middle ear effusion is also noti
ced. The most appropriate next step in the management is to A. advise a tympanos
tomy tube placement B. continue antibiotic prophylaxis with amoxicillin C. presc
ribe a 10-day course of oral antibiotics D. recommend decongestants, along with
oral antibiotics E. start intravenous antibiotic therapy The correct answer is A
. Referrals for discussion of a tympanostomy tube placement should be considered
if there is chronic bilateral effusion for more than 3 months in duration, unil
ateral effusion for more than 3 months in duration, language development delay,
hearing loss of more than

20 decibels, or failure of antibiotic prophylaxis. This particular child has a h


istory of recurrent acute otitis media treated with antibiotic prophylaxis. Fail
ure of antibiotic prophylaxis requires tympanostomy tube placement.
An 8 mo/o infant is braught because of constipation. The father, a "stay-at-home
dad," tells you that she has been having 1 bowel movement every 3-4 days, and t
hat the stool is always very hard. He says that she is doing very well otherwise
; she is a very happy and easy little girl. She is fed primarily infant formula
and he is starting to introduce solid foods. He says that he is concerned becaus
e he remembers always having to change "very dirty" diapers for both of his othe
r children, at least twice a day. Physical examination is unremarkable. A rectal
examination shows guaiac negative brown stool. The most appropriate next step i
s to: A. advise him to give her mineral oil 3 times a day until she is "regular"
B. advise him to give her prune juice or pear juice C. determine thyroid-stimul
ating hormone levels D. order a barium enema E. order rectal manometry and a rec
tal biopsy F. reassure him that all infants have different bowel habits The corr
ect answer is B. Constipation is a common problem in formula-fed infants, and it
is best treated by increasing the amount of fluids in the diet, especially with
fruit juices that contain sorbitol, such as prune and pear, which help to relie
ve constipation. It is often caused by a diet that is too low in fluids or defic
ient of bulk.
A 7 day/o boy who is the product of an uncomplicated gestation is brought to the
physician because of hypospadias. The baby is otherwise healthy, and is urinati
ng without any difficulty. On physical examination, vital signs are stable, lung
s are clear and the heart is beating at a regular rate. The only abnormal physic
al finding is the hypospadias. U/A is negative for infection. Which of the follo
wing is the most appropriate next step? A. Measuring serum creatinine level B. S
chedule a renal ultrasound C. Obtain an IVP D. Cystography E. Performing a circu
mcision

The correct answer is B. Children with hypospadias are prone to urinary tract in
fections and other urinary tract anomalies. They require careful evaluation. A r
enal ultrasound is a safe way of diagnosing neonatal urinary tract pathology.
A 4 y/o old boy falls from the jungle gym at preschool. He sustains minor abrasi
ons and contusions, and is taken care of by the school nurse. His parents take h
im that same afternoon to his regular pediatrician, P/E is unremarkable, his Hgb
=14 g/dL, and U.A shows microhematuria. Which of the following is the most appro
priate next step in management? A. CT scan of the abdomen and pelvis B. Reassure
the parents that microhematuria from minor trauma will resolve spontaneously C.
Serial hemoglobin and hematocrit determinations D. Urologic workup, starting wi
th a sonogram E. Retrograde ureterogram and cystogram Answer is D. Microhematuri
a after trivial trauma in children may be a sign of a congenital anomaly that ma
kes the urinary tract unusually vulnerable. So work up is necessary. The first,
noninvasive test should be the sonogram.
A 5 wk/o infant is brought for a 4 wk hx of noisy breathing that has not improve
d. She has otherwise been healthy except for a current URI for the past 4 days,
which according to the parents, has worsened the noisy breathing. On P/E, she ha
s inspiratory stridor. The noisy breathing improves when the infant is asleep. W
hich of the following is the most likely diagnosis? A. Bronchoalveolar carcinoma
B. Foreign object obstruction C. Laryngomalacia D. Bacterial pneumonia E. Tuber
culosis The correct answer is C. The patient has stridor on examination which is
an inspiratory obstruction that is sensitive to airflow changes. In children, t
he most common cause of stridor is laryngomalacia.
A 5 y/o boy develops a headache, cough, myalgia and a fever. He has been a healt
hy child with all immunizations up to date. He is given a decongestant and an as
pirin for his symptoms with some relief. However, 4 days later, he is brought ba
ck by his parents

because of persistent vomiting and irritability. On P/E, he is found to be semic


omatose, becoming combative on stimulation. Which of the following would confirm
the Dx? A. Serum ammonia B. Serum BUN C. Serum calcium D. Serum opiate level E.
Serum Na The correct answer is A. The child is presenting with symptoms of Reye
syndrome, which is an acute encephalopathy associated with high ammonia levels.
It most commonly occurs in young children after a viral illness. Administration
of aspirin increases the risk of developing this disorder. The vomiting is char
acteristic.
A 3 wk/o African American boy is presented because of a generalized seizure 2 hr
s ago. The infant is highly irritable with high pitched cry, his weight is 2.5 k
g, BP=70 /40 mm Hg, PR=145/min and RR=50/min. Laboratory results: Blood Glc=120
mg/dL BUN= 50 mg/dL Serum Na=170 mEq/L Serum Ca=8.5 mg/dL Serum magnesium=1.5 mg
/dL Which of the following is the most likely cause of this infants seizure? A.
Hypocalcemia B. Hypoglycemia C. Hypomagnesemia D. Intracranial hemorrhage E. Men
ingitis Answer is D- intracranial hemorrhage The level of serum sodium in this p
atient is 170 mEq/L. Infants who have hypernatremic dehydration are irritable an
d lethargic, and have a high-pitched cry. This type of dehydration results from
a greater loss of hypotonic fluid than sodium and accounts for about 15% cases o
f dehydration. Because the patient has no history of diarrhea or vomiting, the h
ypernatremia may be due to inadequate supply of mother s milk that does not matc
h the insensible water loss. Another cause can be the high concentration of sodi
um in mothers milk. Generally, after the child s birth, sodium in the colostrum
decreases from its highest level to its lowest level by the fourth week. However
, some mothers continue to excrete high sodium in their milk and can potentially
cause recurrent hypernatremia and in some case intracranial hemorrhage in the i
nfant.
3 y/o child presents with recurrent Rt. lower lobe pneumonia. Growth parameters
is on 25th percentile & developmentally appropriate for his age. PMH is signific
ant for ear

infection at 18 months & gastroenteritis at 2 yrs of age. Which of the following


conditions is most likely responsible for this pt. s disease? 1.Primary B- or T
- cell immunodeficiency 2.Cystic fibrosis 3.Chediak higashi syn. 4.Congenital lu
ng abnormality 5.Foreign body aspiration Answer is 5. foreign body aspiration Fo
reign body aspiration is the most common cause of recurrent pneumonia in an othe
rwise healthy 3 yo child. Becoz of the lung s anatomy,the rt.side is more common
for aspiration.
A 4 mo/o white male is braught for his well-child visit. Both his father and his
teenage brother have idiopathic epilepsy, which began during early childhood. S
ix hrs after the infant s 2-month DTaP he developed a fever of 39.5 C (103.1 F) re
ctally. His parents are concerned about the safety of further DTP immunization.
Which one of the following is the best approach to DTaP immunization at this vis
it? a. Administer the routine DTaP immunization b. Administer DTaP (acellular pe
rtussis) c. Administer one-half the usual dose of DTaP d. Administer dT (no pert
ussis component) e. Delay further DTaP until he is 6 months old Answer is a. Fal
se assumptions regarding contraindications often result in the needless defermen
t of indicated immunizations. The list of contraindications and precautions for
DTaP immunization does not include a previous febrile reaction unless the fever
was greater than 105 F, nor does it include a family history of seizures.
1- 32 y/o man presents with dizziness and syncope when elevates his right arm. S
ubclavian steal syn. 2- 70 y/o man with back pain, increased urinary hydroxyprol
ine and serum alkaline phosphatase. Pagets dis. of the bone 3- 28 y/o woman compl
ains of tremors, nervousness, and weight loss, ECG shows atrial fibrillation. Hy
perthyroidism 4- 62 y/o man with gastric cancer presents with velvety brown thic
kened skin of the body folds. Acanthosis nigricans

5- 9 y/o boy presents with pain in the medial side of the foot, X-ray shows a na
rrow scaphoid. Kohlers dis. 6- 5 y/o child presents with a painful rib swelling,
X-ray shows a rounded osteolytic lesion. Eosinophilic granuloma 7- 62 y/o man ha
d few episodes of complete blindness in one eye which lasted for 30 seconds each
. TIA 8- 50 y/o man with intermittent claudication, impotence, and pain in the b
uttocks. LeRiche syn. 9- 2 y/o child is brought to you with a thin-walled cyst i
n the supraclavicular area. Cystic hygroma 10- 6 months after cardiac surgery, a
58 year old man develops fever, pericarditis, and an elevated ESR. Dresslers syn
. 11- 8 y/o girl is brought to you with fever, cervical adenopathy, conjunctivit
is, and a scarlatiniform rash on the upper chest. Kawasaki dis. 12- 32 y/o woman
presents with severe right lower quadrant pain and a palpable adnexal mass, she
has no fever and her hCG is negative. Ovarian cyst torsion 13- 24 y/o woman dev
elops erythematous skin eruption following a week s course of antibiotic therapy
for UTI. Erythema multiform 14- 62 y/o woman complains of jaw pain when chewing
and diminished vision of her right eye. Temporal arteritis 15- 52 y/o man prese
nts with severe midchest pain followed by bouts of vomiting, chest X-ray shows p
neumomediastinum. Esophageal rupture 16- 73 y/o woman presents with sticky, yell
owish vaginal discharge, the vaginal mucosa is thin and pale. Atrophic vaginitis
17- 9 y/o boy presents with recurrent episodes of abdominal pain and painful sw
elling in the hands. Hereditary angioedema 18- 62 y/o woman presents with severe
joint disease, hepatosplenomegaly, and leg ulcers. Feltys syn. 19- 38 y/o man co
mplains of severe headaches and blurred vision, his jaw is protruding and his to
ngue is enlarged. Acromegaly

20- 62 y/o man presents with a severe, slowly progressive abdominal pain, follow
ed by a bloody diarrhea. Intestinal infarction 21- 45 y/o diabetic woman present
s with numbness over the palmar side of the thumb, the index and middle fingers.
Carpal tunnel syn. 22- 46 y/o man develops paralysis of the proximal and distal
muscles of the leg after recovering from pneumonia. Guillain barre syn

You might also like